Вы находитесь на странице: 1из 47

NURSING PRACTICE II Community Health Nursing and Care of the Mother and Child SITUATIONAL Situation 1 Nurse Minette

te is an Independent Nurse Practitioner following-up referred clients in their respective homes. Here she handles a case of POSTPARTIAL MOTHER AND FAMILY focusing on HOME CARE. 1. Nurse Minette needs to schedule a first home visit to OB client Leah. When is a first home-care visit typically made? A. Within 4 days after discharge B. Within 24 hours after discharge C. Within 1 hour after discharge D. Within 1 week of discharge 2. Leah is developing constipation from being on bed rest. What measures would you suggest she take to help prevent this? A. Eat more frequent small meals instead of three large one daily B. Walk for at least half an hour daily to stimulate peristalsis C. Drink more milk, increased calcium intake prevents constipation D. Drink eight full glasses of fluid such as water daily 3. If you were Minette, which of the following actions would alert you that a new mother is entering a postpartal taking-hold phase? A. She urges the baby to stay awake so that she can breast-feed him or her B. She tells you she was in a lot of pain all during labor C. She says that she has not selected a name for the baby as yet D. She sleeps as if exhausted from the effort of labor 4. At 6-week postpartum visit what should this postpartal mothers fundic height be? A. Inverted and palpable at the cervix B. Six fingerbreadths below umbilicus C. No longer palpable on her abdomen D. One centimeter above the symphysis pubis 5. This postpartal mother wants to loose the weight she gained in pregnancy, so she is reluctant to increase her caloric intake for breast-feeding. By how much should a lactating mother increase her caloric intake during the first 6 months after birth? A. 350 kcal/day B. 500 kcal/day C. 200 kcal/day D. 1000 kcal/day Situation 2 As the CPE is applicable for all professional nurse, the professional growth and development of Nurses with specialties shall be addressed by a Specialty Certification Council. The following questions apply to these special groups of nurses. 6. Which of the following serves as the legal basis and statute authority for the Board of Nursing to promulgate measures to effect the creation of a Specialty Certification Council and promulgate professional development programs for this group of nurse-professionals? A. R.A. 7610 B. P.D. 223 C. R.A. 9173 D. R.A. 7164 7. By force of law, therefore, the PRC-Board of Nursing released Resolution No. 14 Series of 1999 entitled: Adoption of a Nursing Specialty Certification Program and Creation of Nursing Specialty Certification Council. This rule-making power is called: A. Quasi-Judicial Power B. Regulatory Power C. Quasi-Legislative Power D. Executive/Promulgating Power 8. Under the PRC-Board of Nursing Resolution promulgating the adoption of a Nursing Specialty Certification Program and Council, which two (2) of the following serves as the strongest for its enforcement? (a) Advances made in Science and Technology have provided the climate for specialization in almost all aspects of human endeavor; and (b) As necessary consequence, there has emerged a new concept known as globalization which seeks to remove barriers in trade, industry and services imposed by the national

laws of countries all over the world; and (c) Awareness of this development should impel the nursing sector to prepare our people in the services sector to meet the above challenge; and (d) Current trends of specialization in nursing practice recognized by the International Council of Nurses (ICN) of which the Philippines is a member for the benefit of the Filipino in terms of deepening and refining nursing practice and enhancing the quality of nursing care. A. b & c are strong justifications B. a & b are strong justifications C. a & c are strong justifications D. a & d are strong justifications 9. Which of the following IS NOT a correct statement as regards Specialty Certification? A. The Board of Nursing intended to create the Nursing Specialty Certification Program as a means of perpetuating the creation of an elite force of Filipino Nurse Professionals. B. The Board of Nursing shall oversee the administration of the NSCP through the various Nursing Specialty Boards which will eventually be created C. The Board of Nursing at the time exercised their powers under R.A. 7164 in order to adopt the creation of the Nursing Specialty Certification Council and Program D. The Board of Nursing consulted nursing leaders of national nursing associations and other concerned nursing groups which later decided to ask a special group of nurses of the program for nursing specialty Certification. 10. The NSCC was created for the purpose of implementing the Nursing Specialty policy under the direct supervision and stewardship of the Board of Nursing. Who shall comprise the NSCC? A. A Chairperson who is the current President of the APO; a member from the Academe; and the last member coming from the Regulatory Board B. The chairperson and members of the Regulatory Board ipso facto acts as the CPE Council C. A Chairperson, chosen from among the Regulatory Board members; a Vice Chairperson appointed by the BON at-large; two other members also chosen atlarge; and one representing the consumer group; D. A Chairperson who is the President of the Association from the Academe; a member from the Regulatory Board; and the last member coming from the APO Situation 3 Nurse Anna is a new BSN graduate and has just passed her Licensure Examination for Nurses in the Philippines. She has likewise been hired as a new Community Health Nurse in one of the Rural Health Units in their City, which of the following conditions may be acceptable TRUTHS applied to Community Health Nursing Practice. 11. Which of the following is the primary focus of community health nursing practice? A. Cure of illnesses B. Prevention of illnesses C. Rehabilitation back to health D. Promotion of health 12. In community health nursing, which of the following is our unit of service as nurses? A. The community B. The extended members of every family C. The individual members of the Barangay D. The Family 13. A very important part of the Community Health Nursing Assessment Process includes: A. the application of professional judgment in estimating importance of facts to family and community B. evaluation structures and qualifications of health center team C. coordination with other sectors in relation to health concerns D. carrying out nursing procedures as per plan of action 14. In community health nursing it is important to take into account the family health data coupled with an equally important need to perform ocular inspection of the area as activities which are powerful elements of: A. evaluation

B. assessment C. implementation D. planning 15. The initial step in PLANNING process in order to engage in any nursing project or activities at the community level involves A. goal-setting B. monitoring C. evaluation of data D. provision of data Situation 4 Please continue responding as a professional nurse in these other health situations through the following questions. 16. Transmission of HIV from an infected individual to another person occurs: A. Most frequently in nurses with needlesticks B. Only if there is a large viral load in the blood C. Most commonly as a result of sexual contact D. In all infants born to women with HIV infection 17. The medical record of a client reveals a condition in which the fetus cannot pass through the maternal pelvis. The nurse interprets this as: A. Contracted pelvis B. Maternal disproportion C. Cervical insufficiency D. Fetopelvic disproportion 18. The nurse would anticipate a cesarean birth for a client who has which infection present at the onset of labor? A. Herpes-simplex virus B. Human papilloma virus C. Hepatitis D. Toxoplasmosis 19. After a vaginal examination, the nurse determines that the clients fetus is in an occiput posterior position. The nurse would anticipate that the client will have: A. A precipitous birth B. Intense back pain C. Frequent leg cramps D. Nausea and vomiting 20. The rationales for using a prostaglandin gel for a client prior to the induction of labor is to: A. Soften and efface the cervix B. Numb cervical pain receptors C. Prevent cervical lacerations D. Stimulate uterine contractions Situation 5 Nurse Lorena is a Family Planning and Infertility Nurse Specialist and currently attends to FAMILY PLANNING CLIENTS AND INFERTILE COUPLES. The following conditions pertain to meeting the nursing needs of this particular population group. 21. Dina, 17 years old, asks you how a tubal ligation prevents pregnancy. Which would be the best answer? A. Prostaglandins released from the cut fallopian tubes can kill sperm B. Sperm can not enter the uterus because the cervical entrance is blocked. C. Sperm can no longer reach the ova, because the fallopian tubes are blocked D. The ovary no longer releases ova as there is no where for them to go. 22. The Dators are a couple undergoing testing for infertility. Infertility is said to exist when: A. a woman has no uterus B. a woman has no children C. a couple has been trying to conceive for 1 year D. a couple has wanted a child for 6 months 23. Another client named Lilia is diagnosed as having endometriosis. This condition interferes with fertility because: A. endometrial implants can block the fallopian tubes

B. the uterine cervix becomes inflamed and swollen C. the ovaries stop producing adequate estrogen D. pressure on the pituitary leads to decreased FSH levels 24. Lilia is scheduled to have a hysterosalphingogram. Which of the following instructions would you give her regarding this procedure? A. She will not be able to conceive for 3 months after the procedure B. The sonogram of the uterus will reveal any tumors present C. Many women experience mild bleeding as an after effect D. She may feel some cramping when the dye is inserted 25. Lilias cousin on the other hand, knowing nurse Lorenas specialization asks what artificial insemination by donor entails. Which would be your best answer if you were Nurse Lorena? A. Donor sperm are introduced vaginally into the uterus or cervix B. Donor sperm are injected intra-abdominally into each ovary C. Artificial sperm are injected vaginally to test tubal patency D. The husbands sperm is administered intravenously weekly Situation 6 There are other important basic knowledge in the performance of our task as Community Health Nurse in relation to IMMUNIZATION, these include: 26. The correct temperature to store vaccines in a refrigerator is: A. between -4 deg C and +8 deg C B. between 2 deg C and +8 deg C C. between -8 deg C and 0 deg C D. between -8 deg C and +4 deg C 27. Which of the following vaccines is not done by intramuscular (IM) injection? A. Measles vaccine B. DPT C. Hepa-B vaccine D. Tetanus toxoids 28. This vaccine content is derived from RNA recombinants. A. Measles B. Tetanus toxoids C. Hepatitis B vaccines D. DPT 29. This is the vaccine needed before a child reaches one (1) year in order for him/her to qualify as a :fully immunized child. A. DPT B. Measles C. Hepatitis B D. BCG 30. Which of the following dose of tetanus toxoid is given to the mother to protect her infant from neonatal tetanus and likewise provide 10 years protection for the mother? A. Tetanus toxoid 3 B. Tetanus toxoid 2 C. Tetanus toxoid 1 D. Tetanus toxoid 4 Situation 7 Records contain those comprehensive descriptions of patients health conditions and needs and at the same serve as evidences of every nurses accountability in the care giving process. Nursing records normally differ from institution to institution nonetheless they follow similar patterns of meeting needs for specific types of information. The following pertains to documentation/records management. 31. This special form is used when the patient is admitted to the unit. The nurse completes the information in this record particularly his/her basic personal data, current illness, previous health history, health history of the family, emotional profile, environmental history as well as physical assessment together with nursing diagnosis on admission. What do you call this record? A. Nursing Kardex B. Nursing Health History and Assessment Worksheet C. Medicine and Treatment Record D. Discharge Summary

32. These are sheets/forms which provide an efficient and time saving way to record information that must be obtained repeatedly at regular and/or short intervals of time. This does not replace the progress notes; instead this record of information on vital signs, intake and output, treatment, postoperative care, post partum care, and diabetic regimen, etc. This is used whenever specific measurements or observations are needed to be documented repeatedly. What is this? A. Nursing Kardex B. Graphic Flow Sheets C. Discharge Summary D. Medicine and Treatment Record 33. These records show all medications and treatment provided on a repeated basis. What do you call this record? A. Nursing Health History and Assessment Worksheet B. Discharge Summary C. Nursing Kardex D. Medicine and Treatment Record 34. This flip-over card is usually kept in a portable file at the Nurses Station. It has 2-parts: the activity and treatment section and a nursing care plan section. This carries information about basic demographic data, primary medical diagnosis, current orders of the physician to be carried out by the nurse, written nursing care plan, nursing orders, scheduled tests and procedures, safety precautions in patient care and factors related to daily living activities. This record is used in the charge-of-shift reports or during the bedside rounds or walking rounds. What record is this? A. Discharge Summary B. Medicine and Treatment Record C. Nursing Health History and Assessment Worksheet D. Nursing Kardex 35. Most nurses regard this conventional recording of the date, time, and mode by which the patient leaves a healthcare unit but this record includes importantly, directs of planning for discharge that starts soon after the person is admitted to a healthcare institution. It is accepted that collaboration or multidisciplinary involvement (of all members of the health team) in discharge results in comprehensive care. What do you call this? A. Discharge Summary B. Nursing Kardex C. Medicine and Treatment Record D. Nursing Health History and Assessment Worksheet Situation 8 As Filipino Professional Nurses we must be knowledgeable about the Code of Ethics for Filipino Nurse and practice these by heart. The next questions pertain to this Code of Ethics. 36. Which of the following is TRUE about the Code of Ethics of Filipino Nurses? A. The Philippine Nurses Association for being the accredited professional organization was given the privilege to formulate a Code of Ethics for Nurses which the Board of Nursing promulgated B. Code for Nurses was first formulated in 1982 published in the Proceedings of the Third Annual Convention of the PNA House of Delegates C. The present code utilized the Code of Good Governance for the Professions in the Philippines D. Certificates of Registration of registered nurses may be revoked or suspended for violations of any provisions of the Code of Ethics. 37. Based on the Code of Ethics for Filipino Nurses, what is regarded as the hallmark of nursing responsibility and accountability? A. Human rights of clients, regardless of creed and gender B. The privilege of being a registered professional nurses C. Health, being a fundamental right of every individual D. Accurate documentation of actions and outcomes 38. Which of the following nurses behavior is regarded as a violation of the Code of Ethics of Filipino Nurses? A. A nurse withholding harmful information to the family members of a patient

B. A nurse declining commission sent by a doctor for her referral C. A nurse endorsing a person running for congress. D. Nurse Reviewers and/or nurse review center managers who pays a considerable amount of cash for reviewees who would memorize items from the licensure exams and submit these to them after the examination. 39. A nurse should be cognizant that professional programs for specialty certification by the Board of Nursing accredited through the: A. Professional Regulation Commission B. Nursing Specialty Certification Council C. Association of Deans of Philippine Colleges of Nursing D. Philippine Nurse Association 40. Mr. Santos, R.N. works in a nursing home, and he knows that one of his duties is to be an advocate for his patients. Mr. Santos knows a primary duty of an advocate is to; A. act as the patients legal representative B. complete all nursing responsibilities on time C. safeguard the well being of every patient D. maintain the patients right to privacy Situation 9 Nurse Joanna works as an OB-Gyne Nurse and attends to several HIGH-RISK PREGNANCIES: Particular women with preexisting or Newly Acquired illness. The following conditions apply 41. Bernadette is a 22-year old woman. Which condition would make her more prone than others to developing a Candida infection during pregnancy? A. Her husband plays golf 6 days a week B. She was over 35 when she became pregnant C. She usually drinks tomato juice for breakfast D. She has developed gestational diabetes 42. Bernadette develops a deep vein thrombosis following an auto accident and is prescribed heparin sub-Q. What should Joanna educate her about in regard to this? A. Some infants will be born with allergic symptoms to heparin B. Her infant will be born with scattered petechiae on his trunk C. Heparin can cause darkened skin in newborns D. Heparin does not cross placenta and so does not affect a fetus 43. The cousin of Bernadette with sickle-cell anemia alerted Joanna that she may need further instruction on prenatal care. What statement signifies this fact? A. Ive stopped jogging so I dont risk becoming dehydrated. B. I take an iron pill every day to help grow new red blood cells C. I am careful to drink at least eight glasses of fluid every day D. I understand why folic acid is important for red cell formation 44. Bernadette routinely takes acetylsalicylic acid (aspirin) for arthritis. Why should she limit or discontinue this toward the end of pregnancy? A. Aspirin can lead to deep vein thrombosis following birth B. Newborns develop a red rash from salicylate toxicity C. Newbors develop withdrawal headaches from salicylates D. Salicylates can lead to increased maternal bleeding at childbirth 45. Bernadette received a laceration on her leg from her automobile accident. Why are lacerations of lower extremities potentially more serious in pregnant women than others? A. Lacerations can provoke allergic responses because of gonadothropic hormone B. Increased bleeding can occur from uterine pressure on leg veins C. A woman is less able to keep the laceration clean because of her fatigue D. Healing is limited during pregnancy, so these will not heal until after birth. Situation 10 Still in your self-managed Child Health Nursing Clinic, you encounter these cases pertaining to the CARE OF CHILDREN WITH PULMONARY AFFECTIONS. 46. Josie brought her 3 months old child to your clinic because of cough and colds. Which of the following is your primary action? A. Give cotrimoxazole tablet or syrup B. Assess the patient using the chart on management of children with cough C. Refer to the doctor

D. Teach the mother how to count her childs breathing 47. In responding to the care concerns of children with severe disease, referral to the hospital is of the essence especially if the child manifests which of the following? A. Wheezing B. Stop feeding well C. Fast breathing D. Difficulty to awaken 48. Which of the following is the most important responsibility of a nurse in the prevention of unnecessary deaths from pneumonia and other severe diseases? A. Giving antibiotics B. Taking of the temperature of the sick child C. Provision of Careful Assessment D. Weighing of the sick child 49. You were able to identify factors that lead to respiratory problems in the community where your health facility serve. Your primary role therefore in order to reduce morbidity due to pneumonia is to: A. Teach mothers how to recognize early signs and symptoms of pneumonia B. Make home visits to sick children C. Refer cases to hospitals D. Seek assistance and mobilize the BHWs to have a meeting with mothers 50. Which of the following is the principal focus of the CARI program of the Department of Health? A. Enhancement of health team capabilities B. Teach mothers how to detect signs and where to refer C. Mortality reduction through early detection D. Teach other community health workers how to assess patients. Situation 11 You are working as a Pediatric Nurse in you own Child Health Nursing Clinic. The following cases pertain to ASSESSMENT AND CARE OF THE NEWBORN AT RISK conditions. 51. Theresa, a mother with a 2 year old daughter asks, at what age can I be able to take the blood pressure of my daughter as a routine procedure since hypertension is common in the family? Your answer to this is: A. At 2 years you may B. As early as 1 year old C. When shes 3 years old D. When shes 6 years old 52. You typically gag children to inspect the back of their throat. When is it important NOT to elicit a gag reflex? A. when a girl has a geographic tongue B. When a boy has a possible inguinal hernia C. When a child has symptoms of epiglottitis D. When children are under 5 years of age 53. Baby John was given a drug at birth to reverse the effects of a narcotic given to his mother in labor. What drug is commonly used for this: A. Naloxone (Narcan) B. Morphine Sulfate C. Sodium Chloride D. Penicillin G 54. Why are small-for-gestational-age newborns at risks for difficulty maintaining body temperature? A. They do not have as many fat stores as other infants B. They are more active than usual so throw off covers C. Their skin is more susceptible to conduction of cold D. They are preterm so are born relatively small in size 55. Baby John develops hyperbilirubinemia. What is a method used to treat hyperbilirubinemia in a newborn? A. Keeping infants in a warm and dark environment B. Administration of cardiovascular stimulant C. Gentle exercise to stop muscle breakdown

D. Early feeding to speed passage of meconium Situation 12 You are the nurse in the Out-Patient Department and during your shift you encountered multiple childrens condition. The following questions apply. 56. You assessed a child with visible severe wasting, he has: A. edema B. LBM C. Kwashiorkor D. Marasmus 57. Which of the following conditions is NOT true about contraindication to immunization? A. do not give DPT2 or DPT3 to a child who has had convulsions within 3 days of DPT1 B. do not give BCG if the child has known hepatitis C. do not give DPT to a child who has recurrent convulsion or active neurologic disease D. do not give BCG if the child has known AIDS 58. Which of the following statements about immunization is NOT true: A. A child with diarrhea who is due for OPV should receive the OPV and make extra dose on the next visit B. There is no contraindication to immunization if the child is well enough to go home C. There is no contraindication to immunization if the child is well enough to go home and a child should be immunized in the health center before referral are both correct D. A child should be immunized in the health center before referral 59. A child with visible severe wasting or severe palmar pallor may be classified as: A. moderate malnutrition/anemia B. severe malnutrition/anemia C. not very low weight no anemia D. anemia/very low weight 60. A child has some palmar pallor can be classified as: A. moderate anemia/normal weight B. severe malnutrition/anemia C. anemia/very low weight D. not very low weight no anemia Situation 13 Nette, a nurse palpates the abdomen of Mrs. Medina, a primigravida. She is unsure of the date of her last menstrual period. Leopolds Maneuver is done. The obstetrician told that she appears to be 20 weeks pregnant. 61. Nette explains this because the fundus is: A. At the level of the umbilicus, and the fetal heart can be heard with a fetoscope B. 18 cm, and the baby is just about to move C. Is just over the symphisis, and fetal heart cannot be heard D. 28 cm, and fetal heart can be heard with a Doppler 62. In doing Leopolds Maneuver palpation which among the following IS NOT considered a good preparation: A. The woman should lie in a supine position with her knees flexed slightly B. The hands of the nurse should be cold so that abdominal muscles would contract and tighten C. Be certain that your hands are warm (by washing them in warm water first if necessary) D. The woman empties her bladder before palpation 63. In her pregnancy, she experienced fatigue and drowsiness. This probably occurs because: A. of high blood pressure B. she is expressing pressure C. the fetus utilizes her glucose stores and leaves her with a low blood glucose D. of the rapid growth of the fetus 64. The nurse assesses the woman at 20 weeks gestation and expects the woman to report:

A. Spotting related to fetal implantation B. Symptoms of diabetes as human placental lactogen is released C. Feeling fetal kicks D. Nausea and vomiting related HCG production 65. Mrs. Medina comes to you for check-up on June 2, her EDC is June 11, what do you expect during assessment? A. Fundic ht. 2 fingers below xyphoid process, engaged B. Cervix close, uneffaced, FH-midway between the umbilicus and symphysis pubis C. Cervix open, fundic ht. 2 fingers below xyphoid process, floating D. Fundic height at least at the level of the xyphoid process, engaged Situation 14 Please continue responding as a professional nurse in theses varied health situations through the following questions. 66. Which of the following medications would the nurse expect the physiccian to order for recurrent convulsive seizures of a 10-year old child brought to your clinic? A. Phenobarbital B. Nifedipine C. Butorphanol D. Diazepam 67. RhoGAM is given to Rh-negative women to prevent maternal sensitization from occurring. The nurse is aware that in addition to pregnancy, Rh-negative women would also receive this medication after which of the following? A. Unsuccessful artificial insemination procedure B. Blood transfusion after hemorrhage C. Therapeutic or spontaneous abortion D. Head injury from a car accident 68. Which of the following would the nurse include when describing the pathophysiology of gestational diabetes? A. Glucose levels decrease to accommodate fetal growth B. Hypoinsulinemia develops early in the first trimester C. Pregnancy fosters the development of carbohydrate cravings D. There is progressive resistance to the effects of insulin 69. When providing prenatal education to a pregnant woman with asthma, which of the following would be important for the nurse to do? A. Demonstrate how to assess her blood glucose levels B. Teach correct administration of subcutaneous bronchodilators C. Ensure she seeks treatment for any acute exacerbation D. Explain that she should avoid steroids during her pregnancy 70. Which of the following conditions would cause an insulin-dependent diabetic client the most difficulty during her pregnancy? A. Rh incompatibility B. Placenta Previa C. Hyperemesis Gravidarum D. Abruprtion Placenta Situation 15 One important tool a community health nurse uses in the conduct of his/her activities is the CHN Bag. Which of the following BEST DESCRIBES the use of this vital facility for our practice? 71. The community/Public Health Bag is: A. a requirement for home visits B. an essential and indispensable equipment of the community health nurse C. contains basic medications and articles used by the community health nurse D. a tool used by the Community health nurse is rendering effective nursing procedures during a home visit 72. What is the rationale in the use of bag technique during home visits? A. It helps render effective nursing care to clients or other members of the family B. It saves time and effort of the nurse in the performance of nursing procedures C. It should minimize or prevent the spread of infection from individuals to families D. It should not overshadow concerns for the patient 73. Which among the following is important in the use of the bag technique during home

visit? A. Arrangement of the bags contents must be convenient to the nurse B. The bag should contain all necessary supplies and equipment ready for use C. Be sure to thoroughly clean your bag especially when exposed to communicable disease cases D. Minimize if not totally prevent the spread of infection 74. This is an important procedure of the nurse during home visits. A. protection of the CHN bag B. arrangement of the contents of the CHN bag C. cleaning of the CHN bag D. proper hand washing 75. In consideration of the steps in applying the bag technique, which side of the paper lining of the CHN bag is considered clean to make a non-contaminated work area? A. The lower lip B. The outer surface C. The upper tip D. The inside surface Situation 16 As a Community Health Nurse relating with people in different communities, and in the implementation of health programs and projects you experience vividly as well the varying forms of leadership and management from the Barangay Level to the Local Government/Municipal or City Level. 76. The following statements can correctly be made about Organization and management: A. An organization (or company) is people. Values make people persons; values give vitality, meaning and direction to a company. As the people of an organization value, so the company becomes. B. Management is the process by which administration achieves its mission, goals, and objectives. C. Management effectiveness can be measured in terms of accomplishment of the purpose of the organization while management efficiency is measured in terms of the satisfaction of individual motives. D. Management principles are universal therefore, one need not be concerned about peoples, culture, values, traditions and human relations. A. B and C only C. A and D only B. A, B and D only D. B, A and C only 77. Management by Filipino values advocate the consideration of the Filipino goals trilogy according to the Filipino priority-values which are: A. Family goals, national goals, organizational goals B. Organizational goals, national goals, family goals C. National goals, organizational goals, family goals D. Family goals, organizational goals, national goals 78. Since the advocacy for the utilization of Filipino value-system in management has been encouraged, the Nursing sector is no except, management needs to examine Filipino values and discover its positive potentials and harness them to achieve: A. Employee satisfaction B. Organizational commitments, organizational objectives and employee satisfaction C. Employee objectives/satisfaction, commitments and organizational objectives D. Organizational objectives, commitments and employee objective/satisfaction/ 79. The following statements can correctly be made about an effective and efficient community or even agency managerial-leader. A. Considers the achievement and advancement of the organization she/he represents as well as his people B. Considers the recognition of individual efforts toward the realization of organizational goals as well as the welfare of his people C. Considers the welfare of the organization above all other consideration by higher administration D. Considers its own recognition by higher administration for purposes of promotion and prestige. A. Only C and are correct C. B, C and D are correct

B. A, C and D are correct D. Only A and B are correct 80. Whether management at the community or agency level, there are 3 essential types of skills managers must have, these are: A. Human relation skills, technical skills, and cognitive skills B. Conceptual skills, human relation/behavioral skills, and technical skills C. Technical skills, budget and accounting skills, skills in fund-raising D. Manipulative skills, technical skill, resource management skills A. A and D are correct C. A is correct B. B is correct D. C and D are correct Situation 17 You are actively practicing nurse who just finished your Graduate Studies. You learned the value of Research and would like to utilize the knowledge and skills gained in the application of research to Nursing service. The following questions apply to research. 81. Which type of research inquiry investigates the issues of human complexity (e.g. understanding the human expertise)? A. Logical position B. Naturalistic inquiry C. Positivism D. Quantitative Research 82. Which of the following studies is based on quantitative research? A. A study examining the bereavement process in spouses of clients with terminal cancer B. A study exploring factors influencing weight control behavior. C. A study measuring the effects of sleep deprivation on wound healing D. A study examining clients feeling before, during and after a bone marrow Aspiration 83. Which of the following studies is based on qualitative research? A. A study examining clients reactions to stress after open heart surgery B. A study measuring nutrition and weight loss/gain in clients with cancer C. A study examining oxygen levels after endotracheal suctioning D. A study measuring differences in blood pressure before, during and after a Procedure 84. An 85 year old client in a nursing home tells a nurse, I signed the papers for that research study because the doctor was so insistent and I want him to continue taking care of me. Which client right is being violated? A. Right of self determination B. Right to privacy and confidentiality C. Right to full disclosure D. Right not to be harmed 85. A supposition or system of ideas that is proposed to explain a given phenomenon, best defines: A. a paradigm B. a concept C. a theory D. a conceptual framework Situation 18 Nurse Michelle works with a Family Nursing Team in Calbayog Province specifically handling a UNICEF Project for Children. The following conditions pertain to CARE OF THE FAMILIES WITH PRESCHOOLERS. 86. Ronnie asks constant questions. How many does a typical 3-year-old ask in a days time? A. 1,200 or more B. Less than 50 C. 100-200 D. 300-400 87. Ronnie will need to change to a new bed because his baby sister will need Ronnies old crib. What measure would you suggest that his parents take to help decrease sibling rivalry between Ronnie and his new sister? A. Move him to the new bed before the baby arrives B. Explain that new sisters grow up to become best friends

C. Tell him he will have to share with the new baby D. Ask him to get his crib ready for the new baby 88. Ronnies parents want to know how to react to him when he begins to masturbate while watching television. What would you suggest? A. They refuse to allow him to watch television B. They schedule a health check-up for sex-related disease C. They remind him that some activities are private D. They give her timeout when this begins 89. How many words does a typical 12-month-old infant use? A. About 12 words B. Twenty or more words C. About 50 words D. Two, plus mama and papa 90. As a nurse. You reviewed infant safety procedures with Bryans mother. What are two of the most common types of accidents among infants? A. Aspiration and falls B. Falls and auto accidents C. Poisoning and burns D. Drowning and homicide Situation 19 Among common conditions found in children especially among poor communities are ear infections/problems. The following questions apply. 91. A child with ear problem should be assessed for the following, EXCEPT: A. is there any fever? B. Ear discharge C. If discharge is present for how long? D. Ear pain 92. If the child does not have ear problem, using IMCI, what should you as the nurse do? A. Check for ear discharge B. Check for tender swellings behind the ear C. Check for ear pain D. Go to the next question, check for malnutrition 93. A ear discharge that has been present for more than 14 days can be classified as: A. mastoiditis B. chronic ear infection C. acute ear infection D. complicated ear infection 94. An ear discharge that has been present for less than 14 days can be classified as: A. chronic ear infection B. mastoiditis C. acute ear infection D. complicate ear infection 95. If the child has severe classification because of ear problem, what would be the best thing that you as the nurse can do? A. instruct mother when to return immediately B. refer urgently C. give an antibiotic for 5 days D. dry the ear by wicking Situation 20 If the child with diarrhea registers one sign in the pink row and one in the yellow row in the IMCI Chart 96. We can classify the patient as: A. moderate dehydration B. some dehydration C. no dehydration D. severe dehydration 97. The child with no dehydration needs home treatment. Which of the following is not included in the rules for home treatment in this case: A. continue feeding the child B. gives oresol every 4 hours C. know when to return to the health center

D. give the child extra fluids 98. A child who has had diarrhea for 14 days but has no sign of dehydration is classified as: A. severe persistent diarrhea B. dysentery C. severe dysentery D. persistent diarrhea 99. If the child has sunken eyes, drinking eagerly, thirsty and skin pinch goes back slowly, the classification would be: A. no dehydration B. moderate dehydration C. some dehydration D. severe dehydration 100. Carlo has had diarrhea for 5 days. There is no blood in the stool, he is irritable. His eyes are sunken, the nurse offers fluid to Carlo and he drinks eagerly. When the nurse pinched the abdomen, it goes back slowly. How will you classify Carlos illness? A. severe dehydration B. no dehydration C. some dehydration D. moderate dehydration NURSING PRACTICE III Care of Clients with Physiologic and Psychosocial Alterations (Part A) SITUATIONAL Situation 1 Concerted work efforts among members of the surgical team is essential to the success of the surgical procedure. 1. The sterile nurse or sterile personnel touch only sterile supplies and instruments. When there is a need for sterile supply which is not in the sterile field, who hands out these items by opening its outer cover? A. Circulating Nurse B. Anaesthesiologist C. Surgeon D. Nursing Aide 2. The OR team performs distinct roles for one surgical procedure to be accomplished within a prescribed time frame and deliver a standard patient outcome. While the surgeon performs the surgical procedure, who monitors the status of the client like urine output, blood loss? A. Scrub Nurse B. Surgeon C. Anaesthesiologist D. Circulating Nurse 3. Surgery schedules are communicated to the OR usually a day prior to the procedure by the nurse of the floor or ward where the patient is confined. For orthopedic cases, what department is usually informed to be present in the OR? A. Rehabilitation department B. Laboratory department C. Maintenance department D. Radiology department 4. Minimally invasive surgery is very much into technology. Aside from the usual surgical team, who else has to be present when a client undergoes laparoscopic surgery? A. Information technician B. Biomedical technician C. Electrician D. Laboratory technician 5. In massive blood loss, prompt replacement of compatible blood is crucial. What department needs to be alerted to coordinate closely with the patients family for immediate blood component therapy? A. Security Division B. Chaiplaincy C. Social Service Section

D. Pathology department Situation 2 You are assigned in the Orthopedic Ward where clients are complaining of pain in varying degrees upon movement of body parts. 6. Troy is a one day post open reduction and internal fixation (ORIF) of the left hip and is in pain. Which of the following observation would prompt you to call the doctor? A. Dressing is intact but partially soiled B. Left foot is cold to touch and pedal pulse is absent C. Left leg in limited functional anatomic position D. BP 114/78, pulse of 82 beats/minute 7. There is an order of Demerol 50 mg I.M. now and every 6 hours p r n. You injected Demerol at 5 pm. The next dose of Demerol 50 mg I.M. is given: A. When the client asks for the next dose B. When the patient is in severe pain C. At 11 pm D. At 12 pm 8. You continuously evaluate the clients adaptation to pain. Which of the following behaviors indicate appropriate adaptation? A. The client reports pain reduction and decreased activity B. The client denies existence of pain C. The client can distract himself during pain episodes D. The client reports independence from watchers 9. Pain in ortho cases may not be mainly due to the surgery. There might be other factors such as cultural or psychological that influence pain. How can you alter these factors as the nurse? A. Explain all the possible interventions that may cause the client to worry B. Establish trusting relationship by giving his medication on time C. Stay with the client during pain episodes D. Promote clients sense of control and participation in control by listening to his concerns 10. In some hip surgeries, an epidural catheter for Fentanyl epidural analgesia is given. What is your nursing priority care in such a case? A. Instruct client to observe strict bed rest B. Check for epidural catheter drainage C. Administer analgesia through epidural catheter as prescribed D. Assess respiratory rate carefully Situation 3 Records are vital tools in any institution and should be properly maintained for specific use and time. 11. The patients medical record can work as a double edged sword. When can the medical record become the doctors/nurses worst enemy? A. When the record is voluminous B. When a medical record is subpoenaed in court C. When it is missing D. When the medical record is inaccurate, incomplete, and inadequate 12. Disposal of medical records in government hospitals/institutions must be done in close coordination with what agency? A. Department of Interior and Local Government (DILG) B. Metro Manila Development Authority (MMDA) C. Records Management Archives Office (RMAO) D. Department of Health (DOH) 13. In the hospital, when you need the medical record of a discharged patient for research you will request permission through: A. Doctor in charge B. The hospital director C. The nursing service D. Medical records section 14. You readmitted a client who was in another department a month ago. Since you will need the previous chart, from whom do you request the old chart? A. Central supply section B. Previous doctors clinic

C. Department where the patient was previously admitted D. Medical records section 15. Records Management and Archives Office of the DOH is responsible for implementing its policies on record disposal. You know that your institution is covered by this policy if: A. Your hospital is considered tertiary B. Your hospital is in Metro Manila C. It obtained permit to operate from DOH D. Your hospital is PhilHealth accredited Situation 4 In the OR, there are safety protocols that should be followed. The OR nurse should be well versed with all these to safeguard the safety and quality of patient delivery outcome. 16. Which of the following should be given highest priority when receiving patient in the OR? A. Assess level of consciousness B. Verify patient identification and informed consent C. Assess vital signs D. Check for jewelry, gown, manicure, and dentures 17. Surgeries like I and D (incision and drainage) and debridement are relatively short procedures but considered dirty cases. When are these procedures best scheduled? A. Last case B. In between cases C. According to availability of anaesthesiologist D. According to the surgeons preference 18. OR nurses should be aware that maintaining the clients safety is the overall goal of nursing care during the intraoperative phase. As the circulating nurse, you make certain that throughout the procedure A. the surgeon greets his client before induction of anesthesia B. the surgeon and anesthesiologist are in tandem C. strap made of strong non-abrasive materials are fastened securely around the joints of the knees and ankles and around the 2 hands around an arm board. D. Client is monitored throughout the surgery by the assistant anesthesiologist 19. Another nursing check that should not be missed before the induction of general anesthesia is: A. check for presence underwear B. check for presence dentures C. check patients ID D. check baseline vital signs 20. Some lifetime habits and hobbies affect postoperative respiratory function. If your client smokes 3 packs of cigarettes a day for the past 10 years, you will anticipate increased risk for: A. perioperative anxiety and stress B. delayed coagulation time C. delayed wound healing D. postoperative respiratory function Situation 5 Nurses hold a variety of roles when providing care to a perioperative patient. 21. Which of the following role would be the responsibility of the scrub nurse? A. Assess the readiness of the client prior to surgery B. Ensure that the airway is adequate C. Account for the number of sponges, needles, supplies, used during the surgical procedure. D. Evaluate the type of anesthesia appropriate for the surgical client 22. As a perioperative nurse, how can you best meet the safety need of the client after administering preoperative narcotic? A. Put side rails up and ask the client not to get out of bed B. Send the client to OR with the family C. Allow client to get up to go to the comfort room D. Obtain consent form 23. It is the responsibility of the pre-op nurse to do skin prep for patients undergoing

surgery. If hair at the operative site is not shaved, what should be done to make suturing easy and lessen chance of incision infection? A. Draped B. Pulled C. Clipped D. Shampooed 24. It is also the nurses function to determine when infection is developing in the surgical incision. The perioperative nurse should observe for what signs of impending infection? A. Localized heat and redness B. Serosanguinous exudates and skin blanching C. Separation of the incision D. Blood clots and scar tissue are visible 25. Which of the following nursing interventions is done when examining the incision wound and changing the dressing? A. Observe the dressing and type and odor of drainage if any B. Get patients consent C. Wash hands D. Request the client to expose the incision wound Situation 6 Carlo, 16 years old, comes to the ER with acute asthmatic attack. RR is 46/min and he appears to be in acute respiratory distress. 26. Which of the following nursing actions should be initiated first? A. Promote emotional support B. Administer oxygen at 6L/min C. Suction the client every 30 min D. Administer bronchodilator by nebulizer 27. Aminophylline was ordered for acute asthmatic attack. The mother asked the nurse, what is its indication, the nurse will say: A. Relax smooth muscles of the bronchial airway B. Promote expectoration C. Prevent thickening of secretions D. Suppress cough 28. You will give health instructions to Carlo, a case of bronchial asthma. The health instruction will include the following, EXCEPT: A. Avoid emotional stress and extreme temperature B. Avoid pollution like smoking C. Avoid pollens, dust, seafood D. Practice respiratory isolation 29. The asthmatic client asked you what breathing techniques he can best practice when asthmatic attack starts. What will be the best position? A. Sit in high-Fowlers position with extended legs B. Sit-up with shoulders back C. Push on abdomen during exhalation D. Lean forward 30-40 degrees with each exhalation 30. As a nurse, you are always alerted to monitor status asthmaticus who will likely and initially manifest symptoms of: A. metabolic alkalosis B. respiratory acidosis C. respiratory alkalosis D. metabolic acidosis Situation 7 Joint Commission on Accreditation of Hospital Organization (JCAHO) patient safety goals and requirements include the care and efficient use of technology in the OR and elsewhere in the healthcare facility. 31. As the head nurse in the OR, how can you improve the effectiveness of clinical alarm systems? A. Limit suppliers to a few so that quality is maintained B. Implement a regular inventory of supplies and equipment C. Adherence to manufacturers recommendation D. Implement a regular maintenance and testing of alarm systems 32. Overdosage of medication or anesthetic can happen even with the aid of technology like

infusion pumps, sphygmomanometer and similar devices/machines. As a staff, how can you improve the safety of using infusion pumps? A. Check the functionality of the pump before use B. Select your brand of infusion pump like you do with your cellphone C. Allow the technician to set the infusion pump before use D. Verify the flow rate against your computation 33. JCAHOs universal protocol for surgical and invasive procedures to prevent wrong site, wrong person, and wrong procedure/surgery includes the following, EXCEPT: A. Mark the operative site if possible B. Conduct pre-procedure verification process C. Take a video of the entire intra-operative procedure D. Conduct time out immediately before starting the procedure 34. You identified a potential risk of pre-and postoperative clients. To reduce the risk of patient harm resulting from fall, you can implement the following, EXCEPT: A. Assess potential risk of fall associated with the patients medication regimen B. Take action to address any identified risks through Incident Report (IR) C. Allow client to walk with relative to the OR D. Assess and periodically reassess individual clients risk for falling 35. As a nurse, you know you can improve on accuracy of patients identification by 2 patient identifiers, EXCEPT: A. identify the client by his/her wrist tag and verify with family members B. identify client by his/her wrist tag and call his/her by name C. call the client by his/her case and bed number D. call the patient by his/her name and bed number Situation 8 Team efforts is best demonstrated in the OR. 36. If you are the nurse in charge for scheduling surgical cases, what important information do you need to ask the surgeon? A. Who is your internist B. Who is your assistant and anesthesiologist, and what is your preferred time and type of surgery? C. Who are your anesthesiologist, internist, and assistant D. Who is your anesthesiologist 37. In the OR, the nursing tandem for every surgery is: A. Instrument technician and circulating nurse B. Nurse anesthetist, nurse assistant, and instrument technician C. Scrub nurse and nurse anesthetist D. Scrub and circulating nurses 38. While team effort is needed in the OR for efficient and quality patient care delivery, we should limit the number of people in the room for infection control. Who comprise this team? A. Surgeon, anesthesiologist, scrub nurse, radiologist, orderly B. Surgeon, assistants, scrub nurse, circulating nurse, anesthesiologist C. Surgeon, assistant surgeon, anesthesiologist, scrub nurse, pathologist D. Surgeon, assistant surgeon, anesthesiologist, intern, scrub nurse 39. When surgery is on-going, who coordinates the activities outside, including the family? A. Orderly/clerk B. Nurse Supervisor C. Circulating Nurse D. Anesthesiologist 40. The breakdown in teamwork is often times a failure in: A. Electricity B. Inadequate supply C. Leg work D. Communication Situation 9 Colostomy is a surgically created anus. It can be temporary or permanent, depending on the disease condition. 41. Skin care around the stoma is critical. Which of the following is not indicated as a skin care barriers? A. Apply liberal amount of mineral oil to the area

B. Use karaya paste and rings around the stoma C. Clean the area daily with soap and water before applying bag D. Apply talcum powder twice a day 42. What health instruction will enhance regulation of a colostomy (defecation) of clients? A. Irrigate after lunch everyday B. Eat fruits and vegetables in all three meals C. Eat balanced meals at regular intervals D. Restrict exercise to walking only 43. After ileostomy, which of the following condition is NOT expected? A. Increased weight B. Irritation of skin around the stoma C. Liquid stool D. Establishment of regular bowel movement 44. The following are appropriate nursing interventions during colostomy irrigation, EXCEPT: A. Increase the irrigating solution flow rate when abdominal cramps is felt B. Insert 2-4 inches of an adequately lubricated catheter to the stoma C. Position client in semi-Fowler D. Hang the solution 18 inches above the stoma 45. What sensation is used as a gauge so that patients with ileostomy can determine how often their pouch should be drained? A. Sensation of taste B. Sensation of pressure C. Sensation of smell D. Urge to defecate Situation 10 As a beginner in research, you are aware that sampling is an essential elements of the research process. 46. What does a sample group represent? A. Control group B. Study subjects C. General population D. Universe 47. What is the most important characteristic of a sample? A. Randomization B. Appropriate location C. Appropriate number D. Representativeness 48. Random sampling ensures that each subject has: A. Been selected systematically B. An equal chance of selection C. Been selected based on set criteria D. Characteristics that match other samples 49. Which of the following methods allows the use of any group of research subject? A. Purposive B. Convenience C. Snow-ball D. Quota 50. You decided to include 5 barangays in your municipality and chose a sampling method that would get representative samples from each barangay. What should be the appropriate method ofor you to use in this care? A. Cluster sampling B. Random sampling C. Startified ampling D. Systematic sampling Situation 11 After an abdominal surgery, the circulating and scrub nurses have critical responsibility about sponge and instrument count. 51. When is the first sponge/instrument count reported? A. Before closing the subcutaneous layer B. Before peritoneum is closed C. Before closing the skin

D. Before the fascia is sutured 52. What major supportive layer of the abdominal wall must be sutured with long tensile strength such as cotton or nylon or silk suture? A. Fascia B. Muscle C. Peritoneum D. Skin 53. Like sutures, needles also vary in shape and uses. If you are the scrub nurse for a patient who is prone to keloid formation and has low threshold of pain, what needle would you prepare? A. Round needle B. Atraumatic needle C. Reverse cutting needle D. Tapered needle 54. Another alternative suture for skin closure is the use of ____________ A. Staple B. Therapeutic glue C. Absorbent dressing D. Invisible suture 55. Like any nursing interventions, counts should be documented. To whom does the scrub nurse report any discrepancy of counts so that immediate and appropriate action is instituted? A. Anesthesiologist B. Surgeon C. OR nurse supervisor D. Circulating nurse Situation 12 As a nurse, you should be aware and prepared of the different roles you play. 56. What role do you play when you hold all clients information entrusted to you in the strictest confidence? A. Patients advocate B. Educator C. Patients Liaison D. Patients arbiter 57. As a nurse, you can help improve the effectiveness of communication among healthcare givers by: A. Use of reminders of what to do B. Using standardized list of abbreviations, acronyms, and symbols C. One-on-one oral endorsement D. Text messaging and e-mail 58. As a nurse, your primary focus in the workplace is the clients safety. However, personal safety is also a concern. You can communicate hazards to your co-workers through the use of the following EXCEPT: A. Formal training B. Posters C. Posting IR in the bulletin board D. Use of labels and signs 59. As a nurse, what is one of the best way to reconcile medications across the continuum of care? A. Endorse on a case-to-case basis B. Communicate a complete list of the patients medication to the next provider of service C. Endorse in writing D. Endorse the routine and stat medications every shift 60. As a nurse, you protect yourself and co-workers from misinformation and misrepresentations through the following EXCEPT: A. Provide information to clients about a variety of services that can help alleviate the clients pain and other conditions B. Advising the client, by virtue of your expertise, that which can contribute to the clients well-being

C. Health education among clients and significant others regarding the use of chemical disinfectant D. Endorsement thru trimedia to advertise your favorite disinfectant solution Situation 13 You are assigned at the surgical ward and clients have been complaining of post pain at varying degrees. Pain as you know, is very subjective. 61. A one-day postoperative abdominal surgery client has been complaining of severe throbbing abdominal pain described as 9 in a 1-10 pain rating. Your assessment revelas bowel sounds on all quadrants and the dressing is dry and intact. What nursing intervention would you take A. Medicate client as prescribed B. Encourage client to do imagery C. Encourage deep breathing and turning D. Call surgeon stat 62. Pentoxidone 5 mg IV every 8 hours was prescribed for post abdominal pain. Which will be your priority nursing action? A. Check abdominal dressing for possible swelling B. Explain the proper use of PCA to alleviate anxiety C. Avoid overdosing to prevent dependence/tolerance D. Monitor VS, more importantly RR 63. The client complained of abdominal distention and pain. Your nursing intervention that can alleviate pain is: A. Instruct client to go to sleep and relax B. Advice the client to close the lips and avoid deep breathing and talking C. Offer hot and clear soup D. Turn to sides frequently and avoid too much talking 64. Surgical pain might be minimized by which nursing action in the O.R. A. Skill of surgical team and lesser manipulation B. Appropriate preparation for the scheduled procedure C. Use of modern technology in closing the wound D. Proper positioning and draping of clients 65. One very common cause of postoperative pain is: A. Forceful traction during surgery B. Prolonged surgery C. Break in aseptic technique D. Inadequate anesthetic Situation 14 You were on duty at the medical ward when Zeny came in for admission for tiredness, cold intolerance, constipation, and weight gain. Upon examination, the doctors diagnosis was hypothyroidism. 66. Your independent nursing care for hypothyroidism includes: A. administer sedative round the clock B. administer thyroid hormone replacement C. providing a cool, quiet, and comfortable environment D. encourage to drink 6-8 glasses of water 67. As the nurse, you should anticipate to administer which of the following medications to Zeny who is diagnosed to be suffering from hypothyroidism? A. Levothyroxine B. Lidocaine C. Lipitor D. Levophed 68. Your appropriate nursing diagnosis for Zeny who is suffering from hypothyroidism would probably include which of the following? A. Activity intolerance related to tiredness associated with disorder B. Risk to injury related to incomplete eyelid closure C. Imbalance nutrition to hypermetabolism D. Deficient fluid volume related to diarrhea 69. Myxedema coma is a life threatening complication of long standing and untreated hypothyroidism with one of the following characteristics. A. Hyperglycemia

B. Hypothermia C. Hyperthermia D. Hypoglycemia 70. As a nurse, you know that the most common type of goiter is related to a deficiency of: A. thyroxine B. thyrotropin C. iron D. iodine Situation 15 Mrs. Pichay is admitted to your ward. The MD ordered Prepare for thoracentesis this pm to remove excess air from the pleural cavity. 71. Which of the following nursing responsibilities is essential in Mrs. Pichay who will undergo thoracentesis? A. Support and reassure client during the procedure B. Ensure that informed consent has been signed C. Determine if client has allergic reaction to local anesthesia D. Ascertain if chest x-rays and other tests have been prescribed and completed 72. Mrs. Pichay who is for thoracentesis is assigned by the nurse to which of the following positions? A. Trendelenburg position B. Supine position C. Dorsal Recumbent position D. Orthopneic position 73. During thoracentesis, which of the following nursing intervention will be most crucial? A. Place patient in a quiet and cool room B. Maintain strict aseptic technique C. Advice patient to sit perfectly still during needle insertion until it has been withdrawn from the chest D. Apply pressure over the puncture site as soon as the needle is withdrawn 74. To prevent leakage of fluid in the thoracic cavity, how will you position the client after thoracentesis? A. Place flat in bed B. Turn on the unaffected side C. Turn on the affected side D. On bed rest 75. Chest x-ray was ordered after thoracentesis. When your client asks what is the reason for another chest x-ray, you will explain: A. to rule out pneumothorax B. to rule out any possible perforation C. to decongest D. to rule out any foreign body Situation 16 In the hospital, you are aware that we are helped by the use of a variety of equipment / devices to enhance quality patient care delivery. 76. You are to initiate an IV line to your patient, Kyle, 5, who is febrile. What IV administration set will you prepare? A. Blood transfusion set B. Macroset C. Volumetric chamber D. Microset 77. Kyle is diagnosed to have measles. What will your protective personal attire include? A. Gown B. Eyewear C. Face mask D. Gloves 78. What will you do to ensure that Kyle, who is febrile, will have a liberal oral fluid intake? A. Provide a glass of fruit juice every meal B. Regulate his IV to 30 drops per minute C. Provide a calibrated pitcher of drinking water and juice at the bedside and monitor intake and output

D. Provide a writing pad to record his intake 79. Before bedtime, you went to ensure Kyles safety in bed. You will do which of the following: A. Put the lights on B. Put the side rails up C. Test the call system D. Lock the doors 80. Kyles room is fully mechanized. What do you teach the watcher and Kyle to alert the nurses for help? A. How to lock side rails B. Number of the telephone operator C. Call system D. Remote control Situation 17 Tony, 11 years old, has kissing tonsils and is scheduled for tonsillectomy and adenoidectomy or T and A. 81. You are the nurse of Tony who will undergo T and A in the morning. His mother asked you if Tony will be put to sleep. Your teaching will focus on: A. spinal anesthesia B. anesthesiologists preference C. local anesthesia D. general anesthesia 82. Mothers of children undergoing tonsillectomy and adenoidectomy usually ask what food to prepare and give their children after surgery. You as the nurse will say: A. balanced diet when fully awake B. hot soup when awake C. ice cream when fully awake D. soft diet when fully awake 83. The RR nurse should monitor for the most common postoperative complication of: A. hemorrhage B. endotracheal tube perforation C. osopharyngeal edema D. epiglottis 84. The PACU nurse will maintain postoperative T and A client in what position? A. Supine with neck hyperextended and supported with pillow B. Prone with the head on pillow and turned to the side C. Semi-fowlers with neck flexed D. Reverse trendelenburg with extended neck 85. Tony is to be discharged in the afternoon of the same day after tonsillectomy and adenoidectomy. You as the RN will make sure that the family knows to: A. offer osterized feeding B. offer soft foods for a week to minimize discomfort while swallowing C. supplement his diet with Vitamin C rich juices to enhance healing D. offer clear liquid for 3 days to prevent irritation Situation 18 Rudy was diagnosed to have chronic renal failure. Hemodialysis is ordered so that an A-V shunt was surgically created. 86. Which of the following action would be of highest priority with regards to the external shunt? A. Avoid taking BP or blood sample from the arm with the shunt B. Instruct the client not to exercise the arm with the shunt C. Heparinize the shunt daily D. Change dressing of the shunt daily 87. Diet therapy for Rudy, who has acute renal failure is low-protein, low potassium and low sodium. The nutrition instructions should include: A. Recommend protein of high biologic value like eggs, poultry and lean meats B. Encourage client to include raw cucumbers, carrot, cabbage, and tomatoes C. Allowing the client cheese, canned foods and other processed food D. Bananas, cantaloupe, orange and other fresh fruits can be included in the diet 88. Rudy undergoes hemodialysis for the first time and was scared of disequilibrium

syndrome. He asked you how this can be prevented. Your response is: A. maintain a conducive comfortable and cool environment B. maintain fluid and electrolyte balance C. initial hemodialysis shall be done 30 minutes only so as not to rapidly remove the waste from the blood than from the brain D. maintain aseptic technique throughout the hemodialysis 89. You are assisted by a nursing aide with the care of the client with renal failure. Which delegated function to the aide would you particularly check? A. Monitoring and recording I and O B. Checking bowel movement C. Obtaining vital signs D. Monitoring diet 90. A renal failure patient was ordered for creatinine clearance. As the nurse you will collect A. 48 hour urine specimen B. first morning urine C. 24 hour urine specimen D. random urine specimen Situation 19 Fe is experiencing left sharp pain and occasional hematuria. She was advised to undergo IVP by her physician. 91. Fe was so anxious about the procedure and particularly expressed her low pain threshold. Nursing health instruction will include: A. assure the client that the pain is associated with the warm sensation during the administration of the Hypaque by IV B. assure the client that the procedure painless C. assure the client that contrast medium will be given orally D. assure the client that x-ray procedure like IVP is only done by experts 92. What will the nurse monitor and instruct the client and significant others post IVP? A. Report signs and symptoms for delayed allergic reaction B. Observe NPO for 6 hours C. Increased fluid intake D. Monitor intake and output 93. Post IVP, Fe should excrete the contrast medium. You instructed the family to include more vegetables in the diet and: A. increase fluid intake B. barium enema C. cleansing enema D. gastric lavage 94. The IVP reveals that Fe has small renal calculus that can be passed out spontaneously. To increase the chance of passing the stones, you instructed her to force fluids and do which of the following? A. Balanced diet B. Ambulate more C. Strain all urine D. Bed rest 95. The presence of calculi in the urinary tract is called A. Colelithiasis B. Nephrolithiasis C. Ureterolithiasis D. Urolithiasis Situation 20 At the medical-surgical ward, the nurse must also be concerned about drug interactions. 96. You have a client with TPN. You know that in TPN like blood transfusion, these should be no drug incorporation. However the MDs order read; incorporate insulin to present TPN. Will you follow the order? A. No, because insulin will induce hyperglycemia in patients with TPN B. Yes, because insulin is chemically stable with TPN and can enhance blood glucose level C. No, because insulin is not compatible with TPN

D. Yes, because it was ordered by the MD 97. The RN should also know that some drugs have increased adsorption when infused in the PVC container. How will you administer drugs such as insulin, nitroglycerine hydralazine to promote better therapeutic drug effects? A. Administer by fast drip B. Inject the drugs as close to the IV injection site C. Incorporate to the IV solutions D. Use volumetric chamber 98. One patient had a runaway IV of 50% dextrose. To prevent temporary excess of insulin or transient hyperinsulin reaction what solution you prepare in anticipation of the doctors order? A. Any IV solution available to KVO B. Isotonic solution C. Hypertonic solution D. Hypotonic solution 99. How can nurses prevent drug interaction including adsorption? A. Always flush with NSS after IV administration B. Administering drugs with more diluents C. Improving on preparation techniques D. Referring to manufacturers guidelines 100. In insulin administration, it should be understood that our body normally releases insulin according to our blood glucose level. When is insulin and glucose level highest? A. After excitement B. After a good nights rest C. After an exercise D. After ingestion of food NURSING PRACTICE IV SET B NURSING PRACTICE IV Care of Clients with Physiologic and Psychosocial Alterations (Part B) GENERAL INSTRUCTIONS 1. This test booklet contains 100 test questions 2. Read INSTRUCTIONS TO EXAMINEES printed on your answer sheet. 3. Shade only one (1) box for each question on your answer sheets. Two or more boxes shaded will invalidate your answer. 4. AVOID ERASURES. INSTRUCTIONS 1. Detach one (1) answer sheet from the bottom of your Examinee ID/Answer Sheet Set. 2. Write the subject title Nursing Practice IV on the box provided. 3. Shade Set Box A on your answer sheet if your test booklet is Set A; Set Box B if your test booklet is Set B. MULTIPLE CHOICE Situation 1 Because of the serious consequences of severe burns, management requires a multidisciplinary approach. You have important responsibilities as a nurse. 1. While Sergio was lighting a barbecue grill with a lighter fluid, his shirt burns into flames. The most effective way to extinguish the flames with as little further damage as possible is to: A. log roll on the grass/ground B. slap the flames with his hands C. remove the burning clothes D. pour cold liquid over the flames 2. Once the flames are extinguished, it is most important to: A. cover Sergio with a warm blanket B. give him sips of water C. calculate the extent of his burns D. assess the Sergios breathing 3. Sergio is brought to Emergency Room after the barbecue grill accident. Based on

the assessment of the physician, Sergio sustained superficial partial thickness burns on his trunk, right upper extremities and right lower extremities. His wife asks what that means? Your most accurate response would be: A. Structures beneath the skin are damage B. Dermis is partially damaged C. Epidermis and dermis are both damaged D. Epidermis is damaged 4. During the first 24 hours after the thermal injury, you should asses Sergio for: A. hypokalemia and hypernatremia B. hypokalemia and hyponatremia C. hyperkalemia and hyponatremia D. hyperkalemia and hypernatremia 5. Teddy, who sustained deep partial thickness and full thickness burns of the face, whole anterior chest and both upper extremities two days ago begins to exhibit extreme restlessness. You recognize that this most likely indicates that Teddy is developing: A. Cerebral hypoxia C. metabolic acidosis B. Hypervolemia D. Renal failure Situation 2 You are now working as a staff nurse in a general hospital. You have to be prepared to handle situations with ethico-legal and moral implifications. 6. You are in night duty in surgical ward. One of your patients Martin is a prisoner who sustained an abdominal gunshot wound. He is being guarded by policeman from the local police unit. During your rounds you heard a commotion. You saw the policeman trying to hit Martin. You asked why he was trying to hit Martin. He denied the matter. Which among the following activities will you do first? A. Write an accident report B. Call security officer and report the incident C. Call your nurse supervisor and report the incident D. Call the physician on duty 7. You are on morning duty in the medical ward. You have 10 patients assigned to you. During your endorsement rounds, you found out that one of your patients was not in bed. The patient next to him informed you that he went home without notifying the nurses. Which among the following will you do first? A. Make an incident report B. Call security to report the incident C. Wait for 2 hours before reporting D. Report the incident to your supervisor 8. You are on duty in the medical ward. You were asked to check the narcotics cabinet. You found out that what is on record does not tally with the drugs used. Which among the following will you do first? A. Write an incident report and refer the matter to the nursing director B. Keep your findings to yourself C. Report the matter to your supervisor D. Find out from the endorsement any patient who might have been given narcotics 9. You are on duty in the medical ward. The mother of your patient who is also a nurse, came running to the nurses station and informed you that Fiolo went into cardiopulmonary arrest. A. Start basic life support measures B. Call for the Code C. Bring the crush cart to the room D. Go to see Fiolo and assess for airway patency and breathing problems 10. You are admitting Jorge to the ward and you found out that he is positive for HIV. Which among the following will you do first? A. Take note of it and plan to endorse this to next shift B. Keep this matter to yourself C. Write an incident report D. Report the matter to your head nurse Situation 3 - Colorectal cancer can affect old and younger people. Surgical procedures

and other modes of treatment are done to ensure quality of life. You are assigned in the cancer institute to care of patients with this type of cancer. 11. Larry, 55 years old, who is suspected of having colorectal cancer, is admitted to the CI. After taking the history and vital signs the physician does which test as a screening test for colorectal cancer. A. Barium enema B. Carcinoembryonic antigen C. Annual digital rectal examination D. Proctosigmoidoscopy 12. To confirm his impression of colorectal cancer, Larry will require which diagnostic study? A. Carcinoembryonic antigen B. Proctosigmoidoscopy C. Stool hematologic test D. Abdominal computed tomography (CT) test 13. The following are risk factors for colorectal cancer, EXCEPT: A. Inflammatory bowels B. High fat, high fiber diet C. Smoking D. Genetic factors-familial adenomatous polyposis 14. Symptoms associated with cancer of the colon include: A. constipation, ascites and mucus in the stool B. diarrhea, heart burn and eructation C. blood in the stools, anemia, and pencil shaped stools D. anorexia, hematemesis, and increased peristalasis 15. Several days prior to bowel surgery, Larry may be given sulfasuxidine and neomycin primarily to: A. promote rest of the bowel by minimizing peristalsis B. reduce the bacterial content of the colon C. empty the bowel of solid waste D. soften the stool by retaining water in the colon Situation 4 ENTEROSTOMAL THERAPY is now considered a specialty in nursing. You are participating in the OSTOMY CARE CLASS. 16. You plan to teach Fermin how to irrigate the colostomy when: A. The perineal wound heals And Fermin can sit comfortably on the commode B. Fermin can lie on the side comfortably, about the 3rd postoperative day C. The abdominal incision is closed and contamination is no longer a danger D. The stools starts to become formed, around the 7th postoperative day 17. When preparing to teach Fermin how to irrigate colostomy, you should plan to do the procedure: A. When Fermin would have normal bowel movement B. At least 2 hours before visiting hours C. Prior to breakfast and morning care D. After Fermin accepts alteration in body image 18. When observing a return demonstration of a colostomy irrigation, you know that more teaching is required if Fermin: A. Lubricates the tip of the catheter prior to inserting into the stoma B. Hangs the irrigating bag on the bathroom door cloth hook during fluid insertion C. Discontinues the insertion of fluid after only 500 ml of fluid has been instilled D. Clamps of the flow of fluid when felling uncomfortable 19. You are aware that teaching about colostomy care is understood when Fermin states, I will contact my physician and report: A. If I have any difficulty inserting the irrigating tub into the stoma. B. If I noticed a loss of sensation to touch in the stoma tissue. C. The expulsion of flatus while the irrigating fluid is running out. D. When mucus is passed from the stoma between the irrigations. 20. You would know after teaching Fermin that dietary instruction for him is

effective when he states, It is important that I eat: A. Soft food that are easily digested and absorbed by my large intestines. B. Bland food so that my intestines do not become irritated. C. Food low in fiber so that there is less stool. D. Everything that I ate before the operation, while avoiding foods that cause gas. Situation 5 Ensuring safety is one of your most important responsibilities. You will need to provide instructions and information to your clients to prevent complications. 21. Randy has chest tubes attached to a pleural drainage system. When caring for him you should: A. empty the drainage system at the end of the shift B. clamp the chest tube when suctioning C. palpate the surrounding areas for crepitus D. change the dressing daily using aseptic techniques 22. Fanny, came in from PACU after pelvic surgery. As Fannys nurse you know that the sign that would be indicative of a developing thrombophlebitis would be: A. a tender, painful area on the leg B. a pitting edema of the ankle C. a reddened area at the ankle D. pruritus on the calf and ankle 23. To prevent recurrent attacks on Terry who has acute glumerulonephritis, you should instruct her to: A. seek early treatment for respiratory infections B. take showers instead of tub bath C. continue to take the same restrictions on fluid intake D. avoid situations that involve physical activity 24. Herbert had a laryngectomy and he is now for discharge. He verbalized his concern regarding his laryngectomy tube being dislodged. What should you teach him first? A. Recognize that prompt closure of the tracheal opening may occur B. Keep calm because there is no immediate emergency C. Reinsert another tubing immediately D. Notify the physician at once 25. When caring for Larry after an exploratory chest surgery and pneumonectomy, your priority would be to maintain: A. supplementary oxygen B. ventilation exchange C. chest tube drainage D. blood replacement Situation 6 Infection can cause debilitating consequences when host resistance is compromised and virulence of microorganisms and environmental factors are favorable. Infection control is one important responsibility of the nurse to ensure quality of care. 26. Honrad, who has been complaining of anorexia and feeling tired, develops jaundice, after a workup he is diagnosed of having Hepatitis A. his wife asks you about gamma globulin for herself and her household help. Your most appropriate response would be: A. Dont worry your husbands type of hepatitis is no longer communicable B. Gamma globulin provides passive immunity for hepatitis B C. You should contact your physician immediately about getting gammaglobulin. D. A vaccine has been developed for this type of hepatitis 27. Voltaire develops a nosocomial respiratory tract infection. He ask you what that means? Your best response would be: A. You acquired the infection after you have been admitted to the hospital. B. This is a highly contagious infection requiring complete isolation.

C. The infection you had prior to hospitalization flared up. D. As a result of medical treatment, you have acquired a secondary infection. 28. As a nurse you know that one of the complications that you have to watch out for when caring for Omar who is receiving total parenteral nutrition is: A. stomatitis B. hepatitis C. dysrhythmia D. infection 29. A solution used to treat Pseudomonas wound infection is: A. Dakins solution B. Half-strength hydrogen peroxide C. Acetic acid D. Betadine 30. Which of the following is the most reliable in diagnosing a wound infection? A. Culture and sensitivity B. Purulent drainage from a wound C. WBC count of 20,000/L D. Gram stain testing Situation 7 As a nurse you need to anticipate the occurrence of complications of stroke so that life threatening situations can be prevented. 31. Wendy is admitted to the hospital with signs and symptoms of stroke. Her Glasgow Coma Scale is 6 on admission. A central venous catheter was inserted an I.V. infusion was started. As a nurse assigned to Wendy what will be your priority goal? A. Prevent skin breakdown B. Preserve muscle function C. Promote urinary elimination D. Maintain a patent airway 32. Knowing that for a comatose patient hearing is the last sense to be lost, as Judys nurse, what should you do? A. Tell her family that probably she cant hear them B. Talk loudly so that Wendy can hear you C. Tell her family who are in the room not to talk D. Speak softly then hold her hands gently 33. Which among the following interventions should you consider as the highest priority when caring for June who has hemiparesis secondary to stroke? A. Place June on an upright lateral position B. Perform range of motion exercises C. Apply antiembolic stockings D. Use hand rolls or pillows for support 34. Ivy, age 40, was admitted to the hospital with a severe headache, stiff neck and photophobia. She was diagnosed with a subarachnoid hemorrhage secondary to ruptured aneurysm. While waiting for surgery, you can provide a therapeutic environment by doing which of the following? A. honoring her request for a television B. placing her bed near the window C. dimming the light in her room D. allowing the family unrestricted visiting privileges 35. When performing a neurologic assessment on Walter, you find that his pupils are fixed and dilated. This indicated that he: A. probably has meningitis B. is going to be blind because of trauma C. is permanently paralyzed D. has received a significant brain injury Situation 8 With the improvement in life expectancies and the emphasis in the quality of life it is important to provide quality care to our older patients. There are frequently encountered situations and issues relevant to the older patients. 36. Hypoxia may occur in the older patients because of which of the following

physiologic changes associated with aging. A. Ineffective airway clearance B. Decreased alveolar surfaced area C. Decreased anterior-posterior chest diameter D. Hyperventilation 37. The older patient is at higher risk for incontinence because of: A. dilated urethra B. increased glomerular filtration rate C. diuretic use D. decreased bladder capacity 38. Merle, age 86, is complaining of dizziness when she stands up. This may indicate: A. dementia B. a visual problem C. functional decline D. drug toxicity 39. Cardiac ischemia in an older patient usually produces: A. ST-T wave changes B. Very high creatinine kinase level C. Chest pain radiating to the left arm D. Acute confusion 40. The most dependable sign of infection in the older patient is: A. change in mental status B. fever C. pain D. decreased breath sounds with crackles Situation 9 A disaster is a large-scale emergencyeven a small emergency left unmanaged may turn into a disaster. Disaster preparedness is crucial and is everybodys business. There are agencies that are in charge of ensuring prompt response. Comprehensive Emergency Management (CEM) is an integrated approach to the management of emergency programs and activities for all four emergency phases (mitigation, preparedness, response, and recovery), for all types of emergencies and disasters (natural, man-made, and attack) and for all levels of government and the private sector. 41. Which of the four phases of emergency management is defined as sustained action that reduces or eliminates long-term risk to people and property from natural hazards and their effects.? A. Recovery B. Mitigation C. Response D. Preparedness 42. You are a community health nurse collaborating with the Red Cross and working with disaster relief following a typhoon which flooded and devastated the whole province. Finding safe housing for survivors, organizing support for the family, organizing counseling debriefing sessions and securing physical care are the services you are involved with. To which type of prevention are these activities included: A. Tertiary prevention B. Primary prevention C. Aggregate care prevention D. Secondary prevention 43. During the disaster you see a victim with a green tag, you know that the person: A. has injuries that are significant and require medical care but can wait hours with threat to life or limb B. has injuries that are life threatening but survival is good with minimal intervention C. indicates injuries that are extensive and chances of survival are unlikely even

with definitive care D. has injuries that are minor and treatment can be delayed from hours to days 44. The term given to a category of triage that refers to life threatening or potentially life threatening injury or illness requiring immediate treatment: A. Immediate B. Emergent C. Non-acute D. Urgent 45. Which of the following terms refer to a process by which the individual receives education about recognition of stress reaction and management strategies for handling stress which may be instituted after a disaster? A. Clinical incident stress management B. Follow-up C. Debfriefing D. Defusion Situation 10 As a member of the health and nursing team you have a crucial role to play in ensuring that all the members participate actively is the various tasks agreed upon. 46. While eating his meal, Matthew accidentally dislodges his IV lines and bleeds. Blood oozes on the surface of the over-bed table. It is most appropriate that you instruct the housekeeper to clean the table with: A. Acetone B. Alcohol C. Ammonia D. Bleach 47. You are a member of the infection control team of the hospital. Based on a feedback during the meeting of the committee there is an increased incidence of pseudomonas infection in the Burn Unit (3 out of 10 patients had positive blood and wound culture). What is your priority activity? A. Establish policies for surveillance and monitoring B. Do data gathering about the possible sources of infection (observation, chart review, interview). C. Assign point persons who can implement policies. D. Meet with the nursing group working in the burn unit and discuss problem with them. 48. Part of your responsibility as a member of the diabetes core group is to get referrals from the various wards regarding diabetic patients needing diabetes education. Prior to discharge today, 4 patients are referred to you. How would you start prioritizing your activities? A. Bring your diabetes teaching kit and start your session taking into consideration their distance from your office B. Contact the nurse in-charge and find out from her the reason for the referral C. Determine their learning needs then prioritize D. Involve the whole family in the teaching class 49. You have been designated as a member of the task force to plan activities for the Cancer Consciousness Week. Your committee has 4 months to plan and implement the plan. You are assigned to contact the various cancer support groups in your hospital. What will be your priority activity? A. Find out if there is a budget for this activity B. Clarify objectives of the activity with the task force before contacting the support groups C. Determine the VIPs and Celebrities who will be invited D. Find out how many support groups there are in the hospital and get the contact number of their president 50. You are invited to participate in the medical mission activity of your alumni association. In the planning stage everybody is expected to identify what they can do during the medical mission and what resources are needed. You thought it is also your chance to share what you can do for others. What will

be your most important role where you can demonstrate the impact of nursing in health? A. Conduct health education on healthy life style B. Be a triage nurse C. Take the initial history and document findings D. Act as a coordinator Situation 11 One of the realities that we are confronted with is our mortality. It is important for us nurses to be aware of how we view suffering, pain, illness, and even our death as well as its meaning. That way we can help our patients cope with death and dying. 51. Irma is terminally ill, she speaks to you in confidence. You now feel that Irmas family could be helpful if they knew what Irma has told to you. What should you do first? A. Tell the physician who in turn could tell the family B. Obtain Irmas permission to share the information in the family C. Tell Irma that she has to tell her family what she told you D. Make an appointment to discuss the situation with the family 52. Ruby who has been told she has terminal cancer, turns away and refuses to respond to you. You can best help her by: A. Coming back periodically and indicating your availability if she would like you to sit with her B. Insisting that Ruby should talk with you because it is not good to keep everything inside C. Leaving her alone because she is uncooperative and unpleasant to be with D. Encouraging her to be physically active as possible 53. Leo, who is terminally ill and recognizes that he is in the process of losing everything and everybody he loves, is depressed. Which of the following would best help him during his depression? A. Arrange for visitors who might cheer him B. Sit down and talk with him for a while C. Encourage him to look at the brighter side of things D. Sit silently with him 54. Which of the following statements would best indicate that Ruffy, who is dying, has accepted his impending death? A. Im ready to go. B. I have resigned myself to dying. C. Whats the use? D. Im giving up 55. Marla, 90 years old has planned ahead for her death-philosophically, socially, financially and emotionally. This is recognized as: A. Acceptance that death is inevitable B. Avoidance of the true situation C. Denial with planning for continued life D. Awareness that death will soon occur Situation 12 Brain tumor, whether malignant or benign, has serious management implications. As a nurse, you should be able to understand the consequences of the disease and the treatment. 56. You are caring for Conrad who has a brained tumor and increased Intracranial Pressure (ICP). Which intervention should you include in your plan to reduce ICP? A. Administer bowel softener B. Position Conrad with his head turned toward the side of the tumor C. Provide sensory stimulation D. Encourage coughing and deep breathing 57. Keeping Conrads head and neck alignment results in: A. increased inthrathoracic pressure B. increased venous outflow

C. decreased venous outflow D. increased intrabdominal pressure 58. Which of the following activities may increase intracranial pressure (ICP)? A. Raising the head of the bed B. Manual hyperventilation C. Use of osmotic diuretics D. Valsalvas maneuver 59. After you asses Conrad, you suspected increased ICP. Your most appropriate respiratory goal is to: A. maintain partial pressure of arterial O2 (PaO2) above 80 mmHg B. lower arterial pH C. prevent respiratory alkalosis D. promote CO2 elimination 60. Conrad underwent craniotomy. As his nurse, you know that drainage on a craniotomy dressing must be measured and marked. Which findings should you report immediately to the surgeon? A. Foul-smelling drainage C. Greenish drainage B. Yellowish drainage D. Bloody drainage Situation 13 As a Nurse, you have specific responsibilities as professional. You have to demonstrate specific competencies. 61. The essential components of professional nursing practice are all the following EXCEPT: A. Culture C. Cure B. Care D. Coordination 62. You are assigned to care for four (4) patients. Which of the following patients should you give first priority? A. Grace, who is terminally ill with breast cancer. B. Emy, who was previously lucid but is now unarousable. C. Aris, who is newly admitted and is scheduled for an executive check-up. D. Claire, who has cholelithiasis and is for operation on call. 63. Brenda, the Nursing Supervisor of the intensive care unit (ICU) is not on duty when a staff nurse committed a serious medication error. Which statement accurately reflects the accountability of the nursing supervisor? A. Brenda should be informed when she goes back on duty B. Although Brenda is not on duty, the nursing supervisor on duty decides to call her if time permits C. The nursing supervisor on duty will notify Brenda at home D. Brenda is not duty therefore it is not necessary to inform her. 64. Which barrier should you avoid, to manage time wisely? A. Practical planning B. Procrastination C. Setting limits D. Realistic personal expectation 65. You are caring for Vincent who has just been transferred to the private room. He is anxious because he fears he wont be monitored as closely as he was in the Coronary Care Unit. How can you allay his fear? A. Move his bed to a room far from nurses station to reduce B. Assign the same nurse to him when possible C. Allow Vincent uninterrupted period of time D. Limit Vincents visitors to coincide with CCU policies Situation 14 As a nurse in the Oncology Unit, you have to be prepared to provide safe, efficient and effective care to your patients. 66. Which one of the following nursing interventions would be most helpful in preparing the patient for radiation therapy? A. Offer tranquilizers and antiemetics B. Instruct the patient of the possibility of radiation burn C. Emphasis on the therapeutic value of the treatment D. Map out the precise course of treatment 67. What side effects are most apt to occur the patient during radiation therapy

to the pelvis? A. Urinary retention B. Abnormal vaginal or perineal discharge C. Paresthesia of the lower extremities D. Nausea and vomiting and diarrhea 68. Which of the following can be used on the irradiated skin during a course of radiation therapy? A. Adhesive tape B. Mineral oil C. Talcum powder D. Zinc oxide ointment 69. Earliest sign of skin reaction to radiation therapy is: A. desquamation B. erythema C. atrophy D. pigmentation 70. What is the purpose of wearing a film badge while caring for the patient who is radioactive? A. Identify the nurse who is assigned to care for such a patient B. Prevent radiation-induced sterility C. Protect the nurse from radiation effects D. Measure the amount of exposure to radiation Situation 15 In a disaster there must be a chain of command in place that defined the roles of each member of the response team. Within the health care group there are pre-assigned roles based on education, experience and training on disaster. 71. As a nurse to which of the following groups are you best prepared to join? A. Treatment group C. Morgue management B. Triage group D. Transport group 72. There are important principles that should guide the triage team in disaster management that you have to know if you were to volunteer as part of the triage team. The following principles should be observed in disaster triage, EXCEPT: A. any disaster plan should have resources available to triage at each facility and at the disaster site if possible B. make the most efficient use of available resources C. training on the disaster is not important to the response in the event of a real disaster because each disaster is unique in itself D. do the greatest good for the greatest number of casualties 73. Which of the following categories of conditions should be considered first priority in a disaster? A. Intracranial pressure and mental status B. Lower gastrointestinal problems C. Respiratory infections D. Trauma 74. A guideline that is utilized in determining priorities is to asses the status of the following, EXCEPT: A. perfusion C. respiration B. locomotion D. mentation 75. The most important component of neurologic assessment is: A. pupil reactivity B. vital sign assessment C. cranial nerve assessment D. level of consciousness/responsiveness Situation 16 You are going to participate in a Cancer Consciousness Week. You are assigned to take charge of the women to make them aware of cervical cancer. You reviewed its manifestations and management, 76. The following are risk factors for cervical cancer EXCEPT:

A. immunosuppressive therapy B. sex at an early age, multiple partners, exposure to socially transmitted disease, male partners sexual habits C. viral agents like the Human Papilloma Virus D. smoking 77. Late signs and symptoms of cervical cancer include the following EXCEPT: A. urinary/bowel changes B. pain in pelvis, leg or flank C. uterine bleeding D. lymph edema of lower extremities 78. When a panhysterectomy is performed due to cancer of the cervix, which of the following organs are removed? A. the uterus, cervix, and one ovary B. the uterus, cervix, and two-thirds of the vagina C. the uterus, cervix, tubes and ovaries D. the uterus and cervix 79. The primary modalities of treatment for Stage 1 and IIA cervical cancer include the following: A. surgery, radiation therapy and hormone therapy B. surgery C. radiation therapy D. surgery and radiation therapy 80. A common complication of hysterectomy is A. thrombophelbitis of the pelvic and thigh vessels B. diarrhea due to over stimulation C. atelectasis D. wound dehiscence Situation 17 The body has regulatory mechanism to maintain the needed electrolytes. However there are conditions/surgical interventions that could compromise life. You have to understand how management of these conditions are done. 81. You are caring for Leda who is scheduled to undergo total thyroidectomy because of a diagnosis of thyroid cancer. Prior to total thyroidectomy, you should instruct Leda to: A. Perform range and motion exercise on the head and neck B. Apply gentle pressure against the incision when swallowing C. Cough and deep breath every 2 hours D. Support head with the hands when changing position 82. As Ledas nurse, you plan to set up an emergency equipment at her beside following thyroidectomy. You should include: A. An airway and rebreathing tube B. A tracheostomy set and oxygen C. A crush cart with bed board D. Two ampules of sodium bicarbonate 83. Which of the following nursing interventions is appropriate after a total thyroidectomy? A. Place pillows under your patients shoulders. B. Raise the knee-gatch to 30 degrees C. Keep you patient in a high-fowlers position. D. Support the patients head and neck with pillows and sandbags. 84. If there is an accidental injury to the parathyroid gland during a thyroidectomy which of the following might Leda develop postoperatively? A. Cardiac arrest C. Respiratory failure B. Dyspnea D. Tetany 85. After surgery Leda develops peripheral numbness, tingling and muscle twitching and spasm. What would you anticipate to administer? A. Magnesium sulfate C. Potassium iodide B. Calcium gluconate D. Potassium chloride Situation 18 NURSES are involved in maintaining a safe and healthy

environment. This is part of quality care management. 86. The first step in decontamination is: A. to immediately apply a chemical decontamination foam to the area of contamination B. a through soap and water wash and rinse of the patient C. to immediately apply personal protective equipment D. removal of the patients clothing and jewelry and then rinsing the patient with water 87. For a patient experiencing pruritus, you recommend which type of bath. A. water C. saline B. colloidal (oatmeal D. sodium bicarbonate 88. Induction of vomiting is indicated for the accidental poisoning patient who has ingested. A. rust remover C. toilet bowl cleaner B. gasoline D. aspirin 89. Which of the following term most precisely refer to an infection acquired in the hospital that was not present or incubating at the same time of hospital admission? A. Secondary blood stream infections B. Nosocomial infection C. Emerging infectious disease D. Primary blood stream infection 90. Which of the following guidelines is not appropriate to helping family members cope with sudden death? A. Obtain orders for sedation of family members B. Provide details of the factors attendant to the sudden death C. Show acceptance of the body by touching it and giving the family permission to touch D. Inform the family that the patient has passed on Situation 19 As a nurse you are expected to participate in initiating or participating in the conduct of research studies to improve nursing practice. You have to be updated on the latest trends and issues affecting profession and the best practices arrived at by the profession. 91. You are interested to study the effects of meditation and relaxation on the pain experienced by cancer patients. What type of variable is pain? A. Dependent C. Independent B. Correlational D. Demographic 92. You would like to compare the support system of patients with chronic illness to those with acute illness. How will you best state your problem? A. A Descriptive Study To Compare The Support System Of Patients With Chronic Illness And Those With Acute Illness In Terms Of Demographic Data And Knowledge About Interventions. B. The Effect Of The Type Of Support System of Patients With Chronic Illness And Those With Acute Illness. C. A Comparative Analysis Of The Support System Of Patients With Chronic Illness And Those With Acute Illness. D. A Study To Compare The Support System Of Patients With Chronic Illness And Those With Acute Illness. 93. You would like to compare the support system of patients with chronic illness to those with acute illness. What type of research it this? A. Correlational B. Descriptive C. Experimental D. Quasi-experimental 94. You are shown a Likert Scale that will be used in evaluating your performance in the clinical area. Which of the following questions will you not use in critiquing the Likert Scale? A. Are the techniques to complete and score the scale provided?

B. Are the reliability and validity information on the scale described? C. If the Likert scale is to be used for a study, was the development process described? D. Is the instrument clearly described? 95. In any research study where individual persons are involved, it is important that an informed consent for the study is obtained. The following are essential information about the consent that you should disclose to the prospective subjects EXCEPT: A. Consent to incomplete disclosure B. Descriptions of benefits, risks and discomforts C. Explanation of procedure D. Assurance of anonymity and confidentiality Situation 20 Because severe burn can affect the persons totality it is important that-you apply interventions focusing on the various dimensions of man. You also have to understand the rationale of the treatment. 96. What type of debridement involves proteolytic enzymes? A. Interventional C. Surgical B. Mechanical D. Chemical 97. Which topical antimicrobial is most frequently used in burn wound care? A. Neosporin B. Silver nitrate C. Silver sulfadiazine D. Sulfamylon 98. Hypertrophic burn scars are caused by: A. exaggerated contraction B. random layering of collagen C. wound ischemia D. delayed epithelialization 99. The major disadvantage of whirlpool cleansing of burn wound is: A. patient hypothermia B. cross contamination of wound C. patient discomfort D. excessive manpower requirement 100. Oral analgesics are most frequently used to control burn injury pain: A. upon patient request B. during the emergent phase C. after hospital discharge D. during the acute phase NURSING PRACTICE V- Care of Clients with physiologic and Psychosocial Alterations (Part C). SITUATIONAL Situation 1 Jimmy developed this goal for hospitalization. To get a handle on my nervousness. The nurse is going to collaborate with him to reach his goal. Jimmy was admitted to the hospital because he called his therapist that he planned to asphyxiate himself with exhaust from his car but frightened instead. He realized he needed help. 1. The nurse recognized that Jimmy had conceptualized his problem and the next priority goal in the care plan is: A. help the client find meaning in his experience B. help the client to plan alternatives C. help the client cope with the present problem D. help the client to communicate 2. The nurse is guided that Jimmy is aware of his concerns of the here and now when he crossed out which item from this list of what to know. A. anxiety laden unconscious conflicts

B. subjective idea of the range of mild to severe anxiety C. early signs of anxiety D. physiologic indices of anxiety 3. While Jimmy was discussing the signs and symptoms of anxiety with his nurse, he recognized that complete disruption of the ability to perceive occurs in: A. panic state of anxiety B. severe anxiety C. moderate anxiety D. mild anxiety 4. Jimmy initiates independence and takes an active part in his self care with the following EXCEPT: A. agreeing to contact the staff when he is anxious B. becoming aware of the conscious feeling C. assessing need for medication and medicating himself D. writing out a list of behaviors that he identified as anxious 5. The nurse notes effectiveness of interventions in using subjective and objective data in the: A. initial plans or orders B. database C. problem list D. progress notes Situation 2 A research study was undertaken in order to identify and analyze a disabled boys coping reaction pattern during stress. 6. This study which is an in depth study of one boy is a: A. case study B. longitudinal study C. cross-sectional study D. evaluative study 7. The process recording was the principal tool for data collection. Which of the following is NOT a part of a process recording? A. Non verbal narrative account B. Analysis and interpretation C. Audio-visual recording D. Verbal narrative account 8. Which of these does NOT happen in a descriptive study? A. Exploration of relationship between two or more phenomena. B. Explanation of relationship between two or more phenomena. C. Investigation of a phenomenon in real life context. D. Manipulation of variable 9. The investigator also provided the nursing care of the subject. The investigator is referred to as a/an: A. Participant-observer B. Observer researcher C. Caregiver D. Advocate 10. To ensure reliability of the study, the investigators analysis and interpretations were: A. subjected to statistical treatment B. correlated with a list of coping behaviors C. subjected to an inter-observer agreement D. scored and compared standard criteria Situation 3 During the morning endorsement, the outgoing nurse informed the nursing staff that Regina, 35 years old, was given Flurazepam (Dalmane) 15mg at 10:00pm because she had trouble going to sleep. Before approaching Regina, the nurse read the observation of the night nurse. 11. Which of the following approaches of the nurse validates the data gathered? A. I learned that you were up till ten last night, tell me what happened before you were finally able to sleep and how was your sleep? B. Hmm.. You look like you had a very sound sleep. That pill you were given last night is effective isnt it?

C. Regina, did you sleep well? D. Regina, how are you? 12. Regina is a high school teacher. Which of these information LEAST communicate attention and care for her needs for information about her medicine? A. Guided by a medication teaching plan, go over with her the purpose, indications and special instructions, about the medication and provide her a checklist B. Provide a drug literature. C. Have an informal conversation about the medication and its effects D. Ask her what time she would like to watch the informative video about the medication. 13. The nurse engages Regina in the process of mutual inquiry to provide an opportunity for Regina to: A. face emerging problems realistically B. conceptualize her problem C. cope with her present problem D. perceive her participation in an experience !4. Which of these responses indicate that Regina needs further discussion regarding special instructions? A. I have to take this medicine judiciously B. I know I will stop taking the medicine when there is advice from the doctor for me to discontinue. C. I will inform you and the doctor any untoward reactions I have. D. I like taking this sleeping pill. It solves my problem of insomnia. I wish I can take it for life. 15. Regina commits to herself that she understood and will observe all the medicine precautions by: A. affixing her signature to the teaching plan that she has understood the nurse B. committing what she learned to her memory C. verbally agreeing with the nurse D. relying on her husband to remember the precautions Situation 4 The nurse-patient relationship is a modality through which the nurse meets the clients needs. 16. The nurses most unique tool in working with the emotionally ill client is his/her A. theoretical knowledge B. personality make up C. emotional reactions D. communication skills 17. The psychiatric nurse who is alert to both the physical and emotional needs of clients is working from the philosophical framework that states: A. All behavior is meaningful, communicating a message or a need. B. Human beings are systems of interdependent and interrelated parts. C. Each individual has the potential for growth and change in the direction of positive mental health. D. There is a basic similarity among all human beings. 18. One way to increase objectivity in dealing with ones fears and anxieties is through the process of: A. observation B. intervention C. validation D. collaboration 19. All of the following responses are non therapeutic. Which is the MOST direct violation of the concept, congruence of behavior? A. Responding in a punitive manner to the client. B. Rejecting the client as a unique human being C. Tolerating all behavior in the client. D. Communicating ambivalent messages to the client. 20. The mentally ill person responds positively to the nurse who is warm and caring. This is a demonstration of the nurses role as: A. counselor

B. mother surrogate C. therapist D. socializing agent Situation 5 The nurse engages the client in a nurse-patient interaction. 21. The best time to inform the client about terminating the nurse-patient relationship is: A. when the client asks how long the relationship would be B. during the working phase C. towards the end of the relationship D. at the start of the relationship 22. The client says, I want to tell you something but can you promise that you will keep this a secret? A therapeutic response of the nurse is: A. Yes, our interaction is confidential provided the information you tell me is not detrimental to your safety. B. Of course yes, this is just between you and me. Promise! C. Yes, it is my principle to uphold my clients rights. D. Yes, you have the right to invoke confidentiality of our interaction. 23. When the nurse respects the clients self-disclosure, this is a gauge for the nurses: A. trustworthiness B. loyalty C. integrity D. professionalism 24. Rapport has been established in the nurse-client relationship. The client asks to visit the nurse after his discharge. The appropriate response of the nurse would be: A. The best time to talk is during the nurse-client interaction time. I am committed to have this time available for us while you are at the hospital and ends after your discharge. B. Yes, If you keep it confidential, this is part of privileged communication. C. I am committed for your care. D. I am sorry, though I would want to, it is against hospital policy. 25. The client has not been visited by relatives for months. He gives a telephone number and requests the nurse to call. An appropriate action of the nurse would be: A. Inform the attending physician about the request of the client. B. Assist the client to bring his concern to the attention of the social worker. C. Here (gives her mobile phone). You may call this number now. D. Ask the client what is the purpose of contacting his relatives. Situation 6 Camila, 25 years old, was reported to be gradually withdrawing and isolating herself from friends and family members. She became neglectful of her personal hygiene. She was observed to be talking irrelevantly and incoherently. She was diagnosed as schizophrenia disorder. 26. The past history of Camila would most probably reveal that her premorbid personality is: A. schizoid B. extrovert C. ambivert D. cycloid 27. Camila refuses to relate with to others because she: A. is irritable B. feels superior of others C. anticipates rejection D. is depressed 28. Which of the following disturbances in interpersonal relationships MOST often predispose to the development of schizophrenia? A. Lack of participation in peer groups B. Faulty family atmosphere and interaction C. Extreme rebellion towards authority figures D. Solo parenting 29. Camilas indifference toward the environment is a compensatory behavior to overcome: A. Guilt feelings B. Ambivalence

C. Narcissistic behavior D. Insecurity feelings 30. Schizophrenia is a/an: A. anxiety disorder B. neurosis C. psychosis D. personality disorder Situation 7 Salome, 80 year old widow, has been observed to be irritable, demanding and speaking louder than usual. She would prefer to be alone and take her meals by herself, minimize receiving visitors at home and no longer bothers to answer telephone calls because of deterioration of hearing. She was brought by her daughter to the Geriatric clinic for assessment and treatment. 31. The nurse counsels Salomes daughter that Salomes becoming very loud and tendency to become aggressive is a/an: A. beginning indifference to the world around her B. attempt to maintain authoritative role C. overcompensation for hearing loss D. behavior indicative of unresolved repressed conflict of the past 32. A nursing diagnosis for Salome is: A. sensory deprivation B. social isolation C. cognitive impairment D. ego despair 33. The nurse will assist Salome and her daughter to plan a goal which is for Salome to: A. adjust to the loss of sensory and perceptual function B. participate in conversation and other social situations C. accept the steady loss of hearing that occurs with aging D. increase her self-esteem to maintain her authoritative role 34. The daughter understood, the following ways to assist Salome meet her needs and avoiding which of the following: A. Using short simple sentences B. Speaking distinctly and slowly C. Speaking at eye level and having the clients attention D. Allowing her to take her meals alone 35. Salome was fitted a hearing aid. She understood the proper use and wear of this device when she says that the battery should be functional, the device is turned on and adjusted to a: A. therapeutic level B. comfortable level C. prescribed level D. audible level Situation 8 For more than a month now, Cecilia is persistently feeling restless, worried and feeling as if something dreadful is going to happen. She fears being alone in places and situations where she thinks that no one might come to rescue her just in case something happens to her. 36. Cecilia is demonstrating: A. acrophobia B. claustrophobia C. agoraphobia D. xenophobia 37. Cecilias problem is that she always sees and thinks negative things hence she is always fearful. Phobia is a symptom described as: A. organic B. psychosomatic C. psychotic D. neurotic 38. Cecilia has a lot of irrational thoughts: The goal of therapy is to modify her: A. communication B. cognition

C. observation D. perception 39. Cognitive therapy is indicated for Cecilia when she is already able to handle anxiety reactions. Which of the following should the nurse implement? A. assist her in recognizing irrational beliefs and thoughts B. help find meaning in her behavior C. provide positive reinforcement for acceptable behavior D. Administer anxiolytic drug 40. After discharge, which of these behaviors indicate a positive result of being able to overcome her phobia? A. She reads a book in the public library B. She drives alone along the long expressway. C. She watches television with the family in the recreation room D. She joins an art therapy group Situation 9 It is the first day of clinical experience of nursing students at the Psychiatry Ward. During the orientation, the nurse emphasizes that the team members including nursing students are legally responsible to safeguard patients records from loss or destruction or from people not authorized to read it. 41. It is unethical to tell ones friends and family members data about patients because doing so is a violation of patients rights to: A. Informed consent B. Confidentiality C. Least restrictive environment D. Civil liberty 42. The nurse must see to it that the written consent of mentally ill patients must be taken from: A. Doctor B. Social worker C. Parents or legal guardian D. Law enforcement authorities 43. In an extreme situation and when no other resident or intern is available, should a nurse receive telephone orders, the order has to be correctly written and signed by the physician within: A. 24 hours B. 36 hours C. 48 hours D. 12 hours 44. The following are SOAP (Subjective Objective Analysis Plan) statements on a problem: Anxiety about diagnosis. What is the objective data? A. Relate patients feelings to physician; initiate and encourage her to verbalize her fears; give emotional support by spending more time with patient; continue to make necessary explanations regarding diagnostic tests. B. Has periods of crying; frequently verbalizes fear of what diagnostic tests will reveal C. Anxiety due to unknown D. Im so worried about what else theyll find wrong with me. 45. Nursing care plans provide very meaningful data for the patient profile and initial plan because the focus is on the: A. Summary of chronological notations made by individual health team members B. Identification of patients responses to medical diagnoses and treatment C. Patients responses to health and illness as a total person in interaction with the environment D. Step by step procedures for the management of common problems Situation 10 Marie is 5 years old and described by the mother as bedwetting at night. 46. Which of the following is the MOST common physiological cause of night bed wetting? A. deep sleep factors B. abnormal bladder development C. infections D. familial and genetic factors

47. All of the following, EXCEPT one compromise the concepts of behavior therapy program. A. reward and punishment B. extinction C. learning D. placebo as a form of treatment 48. To help Marie who bed wets at night practice acceptable and appropriate behavior, it is important for the parents to be consistent with the following approaches EXCEPT: A. discipline with a kind attitude B. matter of fact in handling the behavior C. sympathize for the child D. be loving yet firm 49. A therapeutic verbal approach that communicates strong disapproval is: A. You are supposed to get up and go in the toilet when you feel you have to go and did not. The next time you bed wet, Ill tell your friends and hang your sheets out the window for them to see. B. You are supposed to get up and go in the toilet when you feel you have to go and did not. I expect you to from now on without fail. C. If you bed wet, you will change your bed linen and wash the sheets. D. If you dont make an effort to control your bedwetting, Id be upset and disappointed. 50. During your conference, the parent inquires how to motivate Marie to be dry in the morning. Your response which is an immediate intervention would be: A. Give a star each time she wakes up dry and every set of five stars, give a prize. B. Tokens make her materialistic at an early age. Give praise and hugs occasionally. C. What does your child want that you can give every time he/she wakes up dry in the morning? D. Promise him/her a long awaited vacation after school is over. Situation 11 The nurse is often met with the following situations when clients become angry and hostile. 51. To maintain a therapeutic eye contact and body posture while interacting with angry and aggressive individual, the nurse should: A. keep an eye contact while staring at the client B. keep his/her hands behind his/her back or in ones pockets C. fold his/her arms across his/her chest D. keep an open posture, e.g. Hands by sides but palms turned outwards 52. During the pre-interaction phase of the N-P relationship, the nurse recognizes this normal INITIAL reaction to an assaultive or potentially assaultive person. A. To remain and cope with the incident B. Display empathy towards the patient C. To call for help from other members of the team D. To stay and fight or run away 53. Which of the following is an accurate way of reporting and recording an incident? A. When asked about his relationship with his father, client became anxious. B. When asked about his relationship with his father, client clenched his jaw/teeth, made a fist and turned away from the nurse. C. When asked about his relationship with his father, client was resistant to respond D. When asked about his relationship with his father, his anger was suppressed 54. To encourage thought, which of the following approaches is NOT therapeutic? A. Why do you feel angry? B. When do you usually feel angry? C. How do you usually express anger? D. What situations provoke you to be angry? 55. A patient grabs and about to throw it. The nurse best responds saying. A. Stop! Put that chair down. B. Dont be silly. C. Stop! The security will be here in a minute. D. Calm down. Situation 12 Nursing care for the elderly

56. In planning care for a patient with Parkinsons disease, which of these nursing diagnoses should have priority? A. potential for injury B. altered nutritional state C. ineffective coping D. altered mood state 57. A healthy adaptation to aging is primarily related to an individuals A. Number of accomplishments B. Ability to avoid interpersonal conflict C. Physical health throughout life D. Personality development in his life span 58. The frequent use of the older clients name by the nurse is MOST effective in alleviating which of the following responses to old age? A. Loneliness B. Suspicion C. Grief D. Confusion 59. An elderly confused client gets out of bed at night to go to the bathroom and tries to go to another bed when she returns. The MOST appropriate action the nurse would take is to: A. Assign client to a single room B. Leave a light on all night C. Remind client to call the nurse when she wants to get up D. Put side rails on the bed 60. An elderly who has lots of regrets, unhappy and miserable is experiencing: A. Crisis B. Despair C. Loss D. Ambivalence Situation 13 Graciela 1 year old is admitted to the hospital from the emergency room with a fracture of the left femur due to a fall down a flight of stairs. Graciela is placed on Bryants traction. 61. While on Bryants traction, which of these observations of Graciela and her traction apparatus would indicate a decrease in the effectiveness of her traction? A. Gracielas buttocks are resting on the bed. B. The traction weights are hanging 10 inches above the floor. C. Gracielas legs are suspended at a 90 degree angle to her trunk. D. The traction ropes move freely through the pulley. 62. The nurse notes that the fall might also cause a possible head injury. She will be observed for signs of increased intracranial pressure which include: A. Narrowing of the pulse pressure B. Vomiting C. Periorbital edema D. A positive Kernigs sign 63. Graciela is assessed to have no head injury. The Bryants traction is removed. A plaster of Paris hip spica is applied. Which of these finding is a concern of immediate attention that must be reported to the physician immediately? A. Graciela is scratching the cast over her abdomen. B. The toes of Gracielas left foot blanch when the nurse applies pressure on them. C. Gracielas cast is still damp. D. The nurse is unable to insert a finger under the edge of Gracielas cast on her left foot. 64. Part of discharge plan is for the nurse to give instructions about the care of Gracielas cast to the mother. Which of these statements indicate that the mother understood an important aspect of cast care? A. I will use white shoe polish to keep the cast neat. B. I will place plastic sheeting around the perineal area of the cast. C. I will use cool water to wash the cast. D. I will reinforce cracked areas on the cast with adhesive tape.

65. The nurse counsels Gracielas mother ways to safeguard safety while providing opportunities for Graciela to develop a sense of: A. Trust B. Initiative C. Industry D. Autonomy Situation 14 Jolina is an 18 year old beginning college student. Her mother observed that she is having problems relating with her friends. She is undecided about her future. She has lost insight, lost interest in anything and complained of constant tiredness. 66. Jolina is put on antidepressant drugs. These drugs act on the brain chemistry, therefore they would be useful in which type of depression? A. exogenous depression B. neurotic depression C. endogenous depression D. psychotic depression 67. This is a tricyclic antidepressant drug: A. Venlafaxine (Effexor) B. Flouxetine (Prozac) C. Sertraline (Zoloft) D. Imipramine (Tofranil) 68. After one week of antidepressant medication, Jolina still manifests depression. The nurse evaluates this as: A. Unusual because action of antidepressant drug is immediate. B. Unexpected because therapeutic effectiveness takes within a few days. C. Expected because therapeutic effectiveness takes 2-4 weeks. D. Ineffective result because perhaps the drugs dosage is inadequate. 69. Jolina continues to verbalize feeling sad and hopeless. She is not mixing well with other clients. One of the nurses important considerations for Jolina INITIALLY is: A. Formulate a structured schedule so she is able to channel her energies externally B. Let her alone until she feels like mingling with others. C. Encourage her to join socialization hour so she will start to relate with others. D. Encourage her to join group therapy with other patients. 70. During the predischarge conference, the nurse suggests vocational guidance because it should help Jolina to: A. Find a good job. B. Make some decisions about her future C. Realistically assess her assets and limitations D. Solve her own problems Situation 15 Group Approach in Nursing 71. Membership dropout generally occurs in group therapy after a member: A. Accomplishes his goal in joining the group B. Discovers that his feelings are shared by the group members C. Experiences feelings of frustration in the group D. Discusses personal concerns with group members 72. Which of the following questions illustrates the group role of encourager? A. What were you saying? B. Who wants to respond next? C. Where do you go from here? D. Why havent we heard from you? 73. The goal of remotivation therapy is to facilitate: A. Insight B. Productivity C. Socialization D. Intimacy 74. The treatment of the family as a unit is based on the belief that the family: A. is a social system and all the members are interrelated components of that system

B. as a unit of society needs the opportunity to change its own destiny C. who has therapy together will tend to remain together D. is contaminated by the presence of deviant member and all members need treatment. 75. The working phase in a therapy group is usually characterized by which of the following? A. Caution B. Cohesiveness C. Confusion D. Competition Situation 16 The mental health psychiatric nurse functions in a variety of setting with different types of clients. 76. Poverty as reflected in prevalence of communicable diseases, malnutrition and social ills such as street children, homeless and prostitution is a predisposing factor to mental illness. A community approach to cope with this problem is for the nurse to support: A. aggressive family planning methods B. provision of social welfare benefits for the poor C. social action D. free clinics and more hospitals 77. The MOST cost effective way to meet the mental health needs of the public is through programs with a priority goal of: A. treatment B. prevention C. rehabilitation D. research 78. Lorelie upon discharge was referred to a volunteer group where she has learned to read patterns, cut out fabric and use a sewing machine to make simple outfits that will help her earn in the future. What type of therapy is this? A. Recreational therapy B. Art therapy C. Vocational therapy D. Educational therapy 79. In a residential treatment home for adolescent girls, the clients were becoming increasingly tense and upset because of shortening of their recreation time. To deescalate possible anger and aggression among the clients it is BEST to play: A. religious music B. relaxation music C. dance music D. rock music 80. The parents of special children who are behaviorally disturbed need mental health education. Which of these topics would the school nurse consider as priority for their parents class? A. Drug education B. Child abuse C. Effective parenting D. Sex education Situation 17 Nurses in all practice areas are likely to come in contact with clients suffering from acute or chronic drug abuse. 81. The psychodynamic therapy of substance abuse is based upon the premise that drug abuse is: A. a common problem brought about by socioeconomic deprivation B. caused by multiplicity of factors C. predisposed by an inability to develop appropriate psychological resources to manage developmental stresses D. due to biochemical factors 82. Being in contact with reality and the environment is a function of the: A. conscience B. ego C. id D. super ego

83. Substance abuse is different from substance dependence in that, substance dependence: A. includes characteristics of adverse consequences and repeated use B. requires long term treatment in a hospital based program C. produces less severe symptoms than that of abuse D. includes characteristics of tolerance and withdrawal 84. During the detoxification stage, it is a priority for the nurse to: A. teach skills to recognize and respond to health threatening situations B. increase the clients awareness of unsatisfactory protective behaviors C. implement behavior modification D. promote homeostasis and minimize the clients withdrawal symptoms 85. Commonly known as shabu is: A. Cannabis Sativa B. Lysergic acid diethylamide C. Methylenedioxy methamphetamine D. Methamphetamine hydrochloride Situation 18 It is common that clients ask the nurse personal questions. 86. Anticipation of personal questions is given adequate attention during which phase of the nurse patient relationship? A. Orientation phase B. Working phase C. Pre-interaction phase D. Termination phase 87. If the client asks for the nurses telephone number, which of these responses is NOT appropriate? A. It is confidential I just dont give it to anyone. B. What would you do with my number if I give it to you? C. If I say No to your request, what are your thoughts about it? D. Are you asking for an official number of the hospital/clinic for your reference? 88. When the client asks about the family of the nurse, the MOST appropriate response is: A. Avoid the situation and redirect the clients attention B. Give a brief and simple response and focus on the client. C. Why dont we talk about your family instead? D. Introduce another topic like the clients interests 89. When the nurse is asked a personal question, which of these reactions indicates a need for her to introspect? A. The client is simply curious. B. His/Her right to privacy is being intruded. C. The client knows no other way to begin a conversation. D. Some patients are like children in seeking recognition from the nurse. 90. It is 10 oclock on your watch. The client asks, What time is it? The nurses appropriate is: A. Are you getting bored? B. It is 10 oclock. C. Why do you ask? D. Guess, what time is it? Situation 19 Ricky is a 12 year old boy with Downs syndrome. He stands 5 and weighs 100 lbs. he is slim and walks sluggishly with a limp. He wears a neck brace as a support for his neck. X-ray of cervical spine showed subluxation of C1 in relation to C2 with cord compression. He attends a school for a special education. 91. The classroom teacher consults the school nurse for guidance on how to take care of Ricky while inside of the classroom. The nurse considers as priority, Rickys: A. Physiological needs B. Need for self esteem C. Needs for safety and security D. Needs for belonging 92. Rickys mother visited the school nurse. She asked, What should I do when Ricky

fondles his genitalia? An appropriate response of the nurse is for the mother to: A. Divert Rickys attention and engage him in satisfying activities B. Tell Ricky that it is wrong to keep fondling his genitalia C. Ignore Rickys behavior because he will outgrow it later D. Engage him in computer TV games that engage his hands 93. The nurse had one on one health education sessions with Rickys mother. The mother understood that for her son to learn to cope and be independent, she should constantly provide activities for Ricky to be able to: A. socialize with people B. eventually go to school alone C. select and prepare his own food D. do activities of daily living 94. All of the following activities are appropriate for Ricky EXCEPT: A. Working with clay B. Competitive sport C. Preparing and cooking simple menu D. Card and table games 95. Rickys IQ falls within the range of 50-55. he can be expected to: A. Profit from vocational training with moderate supervision B. Live successfully in the community C. Perform simple tasks in closely supervised settings D. Acquire academic skills of 6th grade level Situation 20 The abuse of dangerous drugs is a serious public health concern that nurses need to address. 96. The nurse should recognize that the unit primarily responsible for education and awareness of the members of the family on the ill effects of dangerous drugs is the: A. law enforcement agencies B. school C. church D. family 97. A drug dependent utilizes this defense mechanism and enables him to forget shame and pain. A. repression B. rationalization C. projection D. sublimation 98. This drug produces mirthfulness, fantasies, flight of ideas, loss of train of thought, distortion of size, distance and time, and bloodshot eyes due to dilated pupils. A. Opiates B. LSD C. Marijuana D. Heroin 99. The nurse evaluates that her health teaching to a group of high school boys is effective if these students recognize which of the following dangers of inhalant abuse. A. Sudden death from cardiac or respiratory depression B. Danger of acquiring hepatitis or AIDS C. Experience of blackout D. Psychological dependence after prolonged use 100. The mother of a drug dependent would never consider referring her son to a drug rehabilitation agency because she fears her son might just become worse while relating with other drug users. The mothers behavior can be described as: A. Unhelpful B. Codependent C. Caretaking D. Supportive ***END***

Вам также может понравиться